Slot System
Featured Buckets
Featured Buckets Admin

Vitamin D pearls

Article Type
Changed
Tue, 05/03/2022 - 15:09

Case: A 56-year-old man with a history of type 2 diabetes, hypertension, hyperlipidemia, and obesity calls clinic to discuss concerns about COVID-19, stating: “I want to do everything I can to reduce my risk of infection.” In addition to physical distancing, mask wearing, hand hygiene, and control of chronic conditions, which of the following supplements would you recommend for this patient?

1. Coenzyme Q10 160 mg twice a day

2. Vitamin D 2,000 IU daily

3. Vitamin E 400 IU daily

4. Vitamin B12 1,000 mcg daily

Of these choices, vitamin D supplementation is likely the best option, based on the limited data that is available.

Vitamin D has been implicated in the prevention of many disease processes, including acute respiratory infections. Risk factors for worse COVID-19 outcome, such as older age, obesity, and more pigmented skin are also risk factors for vitamin D deficiency. This makes the study of vitamin D and COVID-19 both challenging and relevant.

In a recent study of 7,807 people living in Israel, Merzon and colleagues found that low plasma vitamin D level was an independent risk factor for COVID-19 infection. Mean plasma vitamin D level was significantly lower among those who tested positive for COVID-19 (19.00 ng/mL) than negative (20.55 ng/ mL). After controlling for demographic variables and several medical conditions, the adjusted odds ratio of COVID-19 infection in those with lower vitamin D was 1.45 (95% confidence interval, 1.08-1.95; P < .001). However, the odds of hospitalization for COVID-19 was not significantly associated with vitamin D level.1

Vitamin D written on a blackboard surrounded by vitamin D dense foods.
yulka3ice/Getty Images

Prior studies have also looked at vitamin D and respiratory infection. Martineau and colleagues analyzed 25 randomized, controlled trials with a pooled number of 11,321 individuals, including healthy ones and those with comorbidities, and found that oral vitamin D supplementation in daily or weekly doses had a protective effect against acute respiratory infection (adjusted odds ratio, 0.88; 95% CI, 0.81-0.96; P < .001). Patients with vitamin D deficiency (less than 25 nmol/L) experienced the most protective benefit. Vitamin D did not influence respiratory infection outcome.2

These studies suggest an adequate vitamin D level may be protective against infection with COVID-19, but who will benefit from vitamin D supplementation, and in what dose? Per U.S. Preventive Services Task Force guidelines, there is insufficient evidence to recommend screening for vitamin D deficiency in asymptomatic adults. Regarding daily dietary intake, the Institute of Medicine recommends 600 IU for persons aged 1-70, and 800 IU for those aged over 70 years. Salmon (447 IU per 3 oz serving), tuna (154 IU), and fortified milk (116 IU) are among the most vitamin D–rich foods.3 The recommended upper level of intake is 4,000 IU/day.
 

Too much of a good thing?

Extra vitamin D is stored in adipose tissue. If it builds up over time, storage sites may be overwhelmed, causing a rise in serum D level. While one might expect a subsequent rise in calcium levels, studies have shown this happens inconsistently, and at very high vitamin D levels, over 120 ng/mL.4 Most people would have to take at least 50,000 IU daily for several months to see an effect. The main adverse outcome of vitamin D toxicity is kidney stones, mediated by increased calcium in the blood and urine.

Ms. Jody Sharninghausen
Jody Sharninghausen

Several animal models have demonstrated hypervitaminosis D–induced aortic and coronary artery calcification. Like with kidney stones, the mechanism appears to be through increased calcium and phosphate levels. Shroff and colleagues studied serum vitamin D levels and vascular disease in children with renal disease on dialysis and found a U-shaped distribution: Children with both low and high vitamin D levels had significantly increased carotid artery intima-media thickness and calcification.5 Given the specialized nature of this population, it’s unclear whether these results can be generalized to most people. More studies are warranted on this topic.
 

Other benefits

Vitamin D is perhaps most famous for helping to build strong bones. Avenell and colleagues performed a Cochrane meta-analysis of vitamin D supplementation in older adults and found that vitamin D alone did not significantly reduce the risk of hip or other new fracture. Vitamin D plus calcium supplementation did reduce the risk of hip fracture (nine trials, pooled number of individuals was 49,853; relative risk, 0.84; P = .01).6

Dr. Douglas S. Paauw, University of Washington, Seattle
Dr. Douglas S. Paauw

A lesser-known benefit of vitamin D is muscle protection. A prospective study out of the Jewish Hospital of Cincinnati followed 146 adults who were intolerant to two or more statins because of muscle side effects and found to have a vitamin D level below 32 ng per mL. Subjects were given vitamin D replacement (50,000 units weekly) and followed for 2 years. On statin rechallenge, 88-95% tolerated a statin with vitamin D levels 53-55 ng/mL.7

Pearl

Vitamin D supplementation may protect against COVID-19 infection and has very low chance of harm at daily doses at or below 4,000 IU. Other benefits of taking vitamin D include bone protection and reduction in statin-induced myopathy. The main adverse effect is kidney stones.

Ms. Sharninghausen is a medical student at the University of Washington, Seattle. Dr. Paauw is professor of medicine in the division of general internal medicine at the University of Washington and serves as third-year medical student clerkship director at the University of Washington. He is a member of the editorial advisory board of Internal Medicine News. Dr. Paauw has no conflicts to disclose. Contact him at imnews@mdedge.com.

References

1. Merzon E et al. Low plasma 25(OH) vitamin D level is associated with increased risk of COVID‐19 infection: An Israeli population‐based study. FEBS J. 2020. doi: 10.1111/febs.15495.

2. Martineau AR et al. Vitamin D supplementation to prevent acute respiratory tract infections: Systematic review and meta-analysis of individual participant data. BMJ. 2017;356:i6583. doi:10.1136/bmj.i6583

3. “How to Get More Vitamin D From Your Food,” Cleveland Clinic. 2019 Oct 23. https://health.clevelandclinic.org/how-to-get-more-vitamin-d-from-your-food/.

4. Galior K et al. Development of vitamin d toxicity from overcorrection of vitamin D Deficiency: A review of case reports. Nutrients. 2018;10(8):953. doi: 10.3390/nu10080953

5. Shroff R et al. A bimodal association of vitamin D levels and vascular disease in children on dialysis. J Am Soc Nephrol. 2008;19(6):1239-46. doi: 10.1681/ASN.2007090993.

6. Avenell A et al. Vitamin D and vitamin D analogues for preventing fractures in post‐menopausal women and older men. Cochrane Database Syst Rev. 2014 Apr 14;2014(4):CD000227. doi: 10.1002/14651858.CD000227.pub4.

7. Khayznikov M et al. Statin intolerance because of myalgia, myositis, myopathy, or myonecrosis can in most cases be safely resolved by vitamin D supplementation. N Am J Med Sci. 2015;7(3):86-93. doi:10.4103/1947-2714.153919
 

Publications
Topics
Sections

Case: A 56-year-old man with a history of type 2 diabetes, hypertension, hyperlipidemia, and obesity calls clinic to discuss concerns about COVID-19, stating: “I want to do everything I can to reduce my risk of infection.” In addition to physical distancing, mask wearing, hand hygiene, and control of chronic conditions, which of the following supplements would you recommend for this patient?

1. Coenzyme Q10 160 mg twice a day

2. Vitamin D 2,000 IU daily

3. Vitamin E 400 IU daily

4. Vitamin B12 1,000 mcg daily

Of these choices, vitamin D supplementation is likely the best option, based on the limited data that is available.

Vitamin D has been implicated in the prevention of many disease processes, including acute respiratory infections. Risk factors for worse COVID-19 outcome, such as older age, obesity, and more pigmented skin are also risk factors for vitamin D deficiency. This makes the study of vitamin D and COVID-19 both challenging and relevant.

In a recent study of 7,807 people living in Israel, Merzon and colleagues found that low plasma vitamin D level was an independent risk factor for COVID-19 infection. Mean plasma vitamin D level was significantly lower among those who tested positive for COVID-19 (19.00 ng/mL) than negative (20.55 ng/ mL). After controlling for demographic variables and several medical conditions, the adjusted odds ratio of COVID-19 infection in those with lower vitamin D was 1.45 (95% confidence interval, 1.08-1.95; P < .001). However, the odds of hospitalization for COVID-19 was not significantly associated with vitamin D level.1

Vitamin D written on a blackboard surrounded by vitamin D dense foods.
yulka3ice/Getty Images

Prior studies have also looked at vitamin D and respiratory infection. Martineau and colleagues analyzed 25 randomized, controlled trials with a pooled number of 11,321 individuals, including healthy ones and those with comorbidities, and found that oral vitamin D supplementation in daily or weekly doses had a protective effect against acute respiratory infection (adjusted odds ratio, 0.88; 95% CI, 0.81-0.96; P < .001). Patients with vitamin D deficiency (less than 25 nmol/L) experienced the most protective benefit. Vitamin D did not influence respiratory infection outcome.2

These studies suggest an adequate vitamin D level may be protective against infection with COVID-19, but who will benefit from vitamin D supplementation, and in what dose? Per U.S. Preventive Services Task Force guidelines, there is insufficient evidence to recommend screening for vitamin D deficiency in asymptomatic adults. Regarding daily dietary intake, the Institute of Medicine recommends 600 IU for persons aged 1-70, and 800 IU for those aged over 70 years. Salmon (447 IU per 3 oz serving), tuna (154 IU), and fortified milk (116 IU) are among the most vitamin D–rich foods.3 The recommended upper level of intake is 4,000 IU/day.
 

Too much of a good thing?

Extra vitamin D is stored in adipose tissue. If it builds up over time, storage sites may be overwhelmed, causing a rise in serum D level. While one might expect a subsequent rise in calcium levels, studies have shown this happens inconsistently, and at very high vitamin D levels, over 120 ng/mL.4 Most people would have to take at least 50,000 IU daily for several months to see an effect. The main adverse outcome of vitamin D toxicity is kidney stones, mediated by increased calcium in the blood and urine.

Ms. Jody Sharninghausen
Jody Sharninghausen

Several animal models have demonstrated hypervitaminosis D–induced aortic and coronary artery calcification. Like with kidney stones, the mechanism appears to be through increased calcium and phosphate levels. Shroff and colleagues studied serum vitamin D levels and vascular disease in children with renal disease on dialysis and found a U-shaped distribution: Children with both low and high vitamin D levels had significantly increased carotid artery intima-media thickness and calcification.5 Given the specialized nature of this population, it’s unclear whether these results can be generalized to most people. More studies are warranted on this topic.
 

Other benefits

Vitamin D is perhaps most famous for helping to build strong bones. Avenell and colleagues performed a Cochrane meta-analysis of vitamin D supplementation in older adults and found that vitamin D alone did not significantly reduce the risk of hip or other new fracture. Vitamin D plus calcium supplementation did reduce the risk of hip fracture (nine trials, pooled number of individuals was 49,853; relative risk, 0.84; P = .01).6

Dr. Douglas S. Paauw, University of Washington, Seattle
Dr. Douglas S. Paauw

A lesser-known benefit of vitamin D is muscle protection. A prospective study out of the Jewish Hospital of Cincinnati followed 146 adults who were intolerant to two or more statins because of muscle side effects and found to have a vitamin D level below 32 ng per mL. Subjects were given vitamin D replacement (50,000 units weekly) and followed for 2 years. On statin rechallenge, 88-95% tolerated a statin with vitamin D levels 53-55 ng/mL.7

Pearl

Vitamin D supplementation may protect against COVID-19 infection and has very low chance of harm at daily doses at or below 4,000 IU. Other benefits of taking vitamin D include bone protection and reduction in statin-induced myopathy. The main adverse effect is kidney stones.

Ms. Sharninghausen is a medical student at the University of Washington, Seattle. Dr. Paauw is professor of medicine in the division of general internal medicine at the University of Washington and serves as third-year medical student clerkship director at the University of Washington. He is a member of the editorial advisory board of Internal Medicine News. Dr. Paauw has no conflicts to disclose. Contact him at imnews@mdedge.com.

References

1. Merzon E et al. Low plasma 25(OH) vitamin D level is associated with increased risk of COVID‐19 infection: An Israeli population‐based study. FEBS J. 2020. doi: 10.1111/febs.15495.

2. Martineau AR et al. Vitamin D supplementation to prevent acute respiratory tract infections: Systematic review and meta-analysis of individual participant data. BMJ. 2017;356:i6583. doi:10.1136/bmj.i6583

3. “How to Get More Vitamin D From Your Food,” Cleveland Clinic. 2019 Oct 23. https://health.clevelandclinic.org/how-to-get-more-vitamin-d-from-your-food/.

4. Galior K et al. Development of vitamin d toxicity from overcorrection of vitamin D Deficiency: A review of case reports. Nutrients. 2018;10(8):953. doi: 10.3390/nu10080953

5. Shroff R et al. A bimodal association of vitamin D levels and vascular disease in children on dialysis. J Am Soc Nephrol. 2008;19(6):1239-46. doi: 10.1681/ASN.2007090993.

6. Avenell A et al. Vitamin D and vitamin D analogues for preventing fractures in post‐menopausal women and older men. Cochrane Database Syst Rev. 2014 Apr 14;2014(4):CD000227. doi: 10.1002/14651858.CD000227.pub4.

7. Khayznikov M et al. Statin intolerance because of myalgia, myositis, myopathy, or myonecrosis can in most cases be safely resolved by vitamin D supplementation. N Am J Med Sci. 2015;7(3):86-93. doi:10.4103/1947-2714.153919
 

Case: A 56-year-old man with a history of type 2 diabetes, hypertension, hyperlipidemia, and obesity calls clinic to discuss concerns about COVID-19, stating: “I want to do everything I can to reduce my risk of infection.” In addition to physical distancing, mask wearing, hand hygiene, and control of chronic conditions, which of the following supplements would you recommend for this patient?

1. Coenzyme Q10 160 mg twice a day

2. Vitamin D 2,000 IU daily

3. Vitamin E 400 IU daily

4. Vitamin B12 1,000 mcg daily

Of these choices, vitamin D supplementation is likely the best option, based on the limited data that is available.

Vitamin D has been implicated in the prevention of many disease processes, including acute respiratory infections. Risk factors for worse COVID-19 outcome, such as older age, obesity, and more pigmented skin are also risk factors for vitamin D deficiency. This makes the study of vitamin D and COVID-19 both challenging and relevant.

In a recent study of 7,807 people living in Israel, Merzon and colleagues found that low plasma vitamin D level was an independent risk factor for COVID-19 infection. Mean plasma vitamin D level was significantly lower among those who tested positive for COVID-19 (19.00 ng/mL) than negative (20.55 ng/ mL). After controlling for demographic variables and several medical conditions, the adjusted odds ratio of COVID-19 infection in those with lower vitamin D was 1.45 (95% confidence interval, 1.08-1.95; P < .001). However, the odds of hospitalization for COVID-19 was not significantly associated with vitamin D level.1

Vitamin D written on a blackboard surrounded by vitamin D dense foods.
yulka3ice/Getty Images

Prior studies have also looked at vitamin D and respiratory infection. Martineau and colleagues analyzed 25 randomized, controlled trials with a pooled number of 11,321 individuals, including healthy ones and those with comorbidities, and found that oral vitamin D supplementation in daily or weekly doses had a protective effect against acute respiratory infection (adjusted odds ratio, 0.88; 95% CI, 0.81-0.96; P < .001). Patients with vitamin D deficiency (less than 25 nmol/L) experienced the most protective benefit. Vitamin D did not influence respiratory infection outcome.2

These studies suggest an adequate vitamin D level may be protective against infection with COVID-19, but who will benefit from vitamin D supplementation, and in what dose? Per U.S. Preventive Services Task Force guidelines, there is insufficient evidence to recommend screening for vitamin D deficiency in asymptomatic adults. Regarding daily dietary intake, the Institute of Medicine recommends 600 IU for persons aged 1-70, and 800 IU for those aged over 70 years. Salmon (447 IU per 3 oz serving), tuna (154 IU), and fortified milk (116 IU) are among the most vitamin D–rich foods.3 The recommended upper level of intake is 4,000 IU/day.
 

Too much of a good thing?

Extra vitamin D is stored in adipose tissue. If it builds up over time, storage sites may be overwhelmed, causing a rise in serum D level. While one might expect a subsequent rise in calcium levels, studies have shown this happens inconsistently, and at very high vitamin D levels, over 120 ng/mL.4 Most people would have to take at least 50,000 IU daily for several months to see an effect. The main adverse outcome of vitamin D toxicity is kidney stones, mediated by increased calcium in the blood and urine.

Ms. Jody Sharninghausen
Jody Sharninghausen

Several animal models have demonstrated hypervitaminosis D–induced aortic and coronary artery calcification. Like with kidney stones, the mechanism appears to be through increased calcium and phosphate levels. Shroff and colleagues studied serum vitamin D levels and vascular disease in children with renal disease on dialysis and found a U-shaped distribution: Children with both low and high vitamin D levels had significantly increased carotid artery intima-media thickness and calcification.5 Given the specialized nature of this population, it’s unclear whether these results can be generalized to most people. More studies are warranted on this topic.
 

Other benefits

Vitamin D is perhaps most famous for helping to build strong bones. Avenell and colleagues performed a Cochrane meta-analysis of vitamin D supplementation in older adults and found that vitamin D alone did not significantly reduce the risk of hip or other new fracture. Vitamin D plus calcium supplementation did reduce the risk of hip fracture (nine trials, pooled number of individuals was 49,853; relative risk, 0.84; P = .01).6

Dr. Douglas S. Paauw, University of Washington, Seattle
Dr. Douglas S. Paauw

A lesser-known benefit of vitamin D is muscle protection. A prospective study out of the Jewish Hospital of Cincinnati followed 146 adults who were intolerant to two or more statins because of muscle side effects and found to have a vitamin D level below 32 ng per mL. Subjects were given vitamin D replacement (50,000 units weekly) and followed for 2 years. On statin rechallenge, 88-95% tolerated a statin with vitamin D levels 53-55 ng/mL.7

Pearl

Vitamin D supplementation may protect against COVID-19 infection and has very low chance of harm at daily doses at or below 4,000 IU. Other benefits of taking vitamin D include bone protection and reduction in statin-induced myopathy. The main adverse effect is kidney stones.

Ms. Sharninghausen is a medical student at the University of Washington, Seattle. Dr. Paauw is professor of medicine in the division of general internal medicine at the University of Washington and serves as third-year medical student clerkship director at the University of Washington. He is a member of the editorial advisory board of Internal Medicine News. Dr. Paauw has no conflicts to disclose. Contact him at imnews@mdedge.com.

References

1. Merzon E et al. Low plasma 25(OH) vitamin D level is associated with increased risk of COVID‐19 infection: An Israeli population‐based study. FEBS J. 2020. doi: 10.1111/febs.15495.

2. Martineau AR et al. Vitamin D supplementation to prevent acute respiratory tract infections: Systematic review and meta-analysis of individual participant data. BMJ. 2017;356:i6583. doi:10.1136/bmj.i6583

3. “How to Get More Vitamin D From Your Food,” Cleveland Clinic. 2019 Oct 23. https://health.clevelandclinic.org/how-to-get-more-vitamin-d-from-your-food/.

4. Galior K et al. Development of vitamin d toxicity from overcorrection of vitamin D Deficiency: A review of case reports. Nutrients. 2018;10(8):953. doi: 10.3390/nu10080953

5. Shroff R et al. A bimodal association of vitamin D levels and vascular disease in children on dialysis. J Am Soc Nephrol. 2008;19(6):1239-46. doi: 10.1681/ASN.2007090993.

6. Avenell A et al. Vitamin D and vitamin D analogues for preventing fractures in post‐menopausal women and older men. Cochrane Database Syst Rev. 2014 Apr 14;2014(4):CD000227. doi: 10.1002/14651858.CD000227.pub4.

7. Khayznikov M et al. Statin intolerance because of myalgia, myositis, myopathy, or myonecrosis can in most cases be safely resolved by vitamin D supplementation. N Am J Med Sci. 2015;7(3):86-93. doi:10.4103/1947-2714.153919
 

Publications
Publications
Topics
Article Type
Sections
Disallow All Ads
Content Gating
No Gating (article Unlocked/Free)
Alternative CME
Disqus Comments
Default
Use ProPublica
Hide sidebar & use full width
render the right sidebar.
Conference Recap Checkbox
Not Conference Recap
Clinical Edge
Display the Slideshow in this Article

Migraine headache pearls

Article Type
Changed
Thu, 10/08/2020 - 10:06

A 25-year-old woman presents to discuss treatment of her headaches. They occur two or three times a month and last for 4-6 hours. The headaches are disabling, have a pounding quality behind the patient’s right eye, and worsen with exercise. The patient’s neurologic exam is normal.

She has tried oral sumatriptan and naproxen, but neither drug provided her with any relief from the headaches. What treatment would you recommend?
 

A. Topiramate

B. Beta-blocker

C. Lasmiditan

D. Metoclopramide plus sumatriptan

E. Ubrogepant

It is common to see patients with migraine headaches and to see patients with migraines who have not responded to previous migraine therapies. This column will focus on some migraine therapy pearls.

DKart/iStockphoto

For this patient, I would try choice D first, giving metoclopramide with oral sumatriptan to see if it can improve response to sumatriptan. The two new classes of drugs for acute migraine therapy, the gepants and ditans, certainly have a role in patients unresponsive or intolerant of triptans/NSAIDS, but I would try several tricks with these less expensive medications first before entering into prior authorization hell trying to get a gepant or ditan.

When a patient has already used a triptan but experienced no benefit from it, often the next medication a patient tries is a different triptan. Dahlof reviewed four trials that looked at the efficacy of switching sumatriptan nonresponders to a different triptan and found that lack of response to sumatriptan did not predict lack of response to an alternative triptan.1 Unfortunately, acquiring insurance coverage for an alternate triptan can be difficult.

Other treatment options are nasal or injectable formulations of sumatriptan. Both of these are more costly than oral sumatriptan, and injectable sumatriptan has more side effects than oral triptans.

Combining treatment with metoclopramide can be helpful. In a study by Schulman and Dermott looking at patients who had previously been triptan nonresponders, 63% of those who took metoclopramide with sumatriptan had meaningful pain relief, compared with 31% of those who received sumatriptan and placebo.2

In a different study, Tfelt-Hansen et al. compared treatment with the combination of lysine acetylsalicylate plus metoclopramide versus treatment with 100 mg of sumatriptan.3 There was no difference in outcomes between the two treatment groups, with the lysine acetylsalicylate plus metoclopramide patients having a 57% success rate for first treated migraine compared with 53% of the sumatriptan-treated patients.

Dr. Douglas S. Paauw, University of Washington, Seattle
Dr. Douglas S. Paauw

Treating with the combination of naproxen plus sumatriptan is superior to treating with either medication alone. Brandes et al. reported on two studies involving the use of the sumatriptan/naproxen combination, compared with using sumatriptan, naproxen, or placebo.4 In both, taking the sumatriptan/naproxen combination was superior to taking sumatriptan, naproxen, or placebo (P < .001).

In a study of patients with poor prior response to triptans, Mathew et al. found that the sumatriptan/naproxen combination was superior to placebo for both 2- and 24-hour headache relief (P < .001).5
 

Pearl

Try several options before abandoning triptans in previous triptan nonresponders, including trying a different triptan, adding metoclopramide, orcombining with an NSAID.

Dr. Paauw is professor of medicine in the division of general internal medicine at the University of Washington, Seattle, and he serves as third-year medical student clerkship director at the University of Washington. He is a member of the editorial advisory board of Internal Medicine News. Dr. Paauw has no conflicts to disclose. Contact him at imnews@mdedge.com.

References

1. Dahlöf CG. Infrequent or nonresponse to oral sumatriptan does not predict response to other triptans – review of four trials. Cephalalgia. 2006 Feb;26(2):98-106.

2. Schulman EA, Dermott KF. Sumatriptan plus metoclopramide in triptan-nonresponsive migraineurs. Headache. 2003 Jul-Aug;43(7):729-33.

3. Tfelt-Hansen P et al. The effectiveness of combined oral lysine acetylsalicylate and metoclopramide compared with oral sumatriptan for migraine. Lancet. 1995 Oct 7;346(8980):923-6.

4. Brandes JL et al. Sumatriptan‐naproxen for acute treatment of migraine: A randomized trial. JAMA. 2007;297:1443‐54.

5. Mathew NT, Landy S, Stark S, et al. Fixed‐dose sumatriptan and naproxen in poor responders to triptans with a short half‐life. Headache. 2009;49:971‐82.

Publications
Topics
Sections

A 25-year-old woman presents to discuss treatment of her headaches. They occur two or three times a month and last for 4-6 hours. The headaches are disabling, have a pounding quality behind the patient’s right eye, and worsen with exercise. The patient’s neurologic exam is normal.

She has tried oral sumatriptan and naproxen, but neither drug provided her with any relief from the headaches. What treatment would you recommend?
 

A. Topiramate

B. Beta-blocker

C. Lasmiditan

D. Metoclopramide plus sumatriptan

E. Ubrogepant

It is common to see patients with migraine headaches and to see patients with migraines who have not responded to previous migraine therapies. This column will focus on some migraine therapy pearls.

DKart/iStockphoto

For this patient, I would try choice D first, giving metoclopramide with oral sumatriptan to see if it can improve response to sumatriptan. The two new classes of drugs for acute migraine therapy, the gepants and ditans, certainly have a role in patients unresponsive or intolerant of triptans/NSAIDS, but I would try several tricks with these less expensive medications first before entering into prior authorization hell trying to get a gepant or ditan.

When a patient has already used a triptan but experienced no benefit from it, often the next medication a patient tries is a different triptan. Dahlof reviewed four trials that looked at the efficacy of switching sumatriptan nonresponders to a different triptan and found that lack of response to sumatriptan did not predict lack of response to an alternative triptan.1 Unfortunately, acquiring insurance coverage for an alternate triptan can be difficult.

Other treatment options are nasal or injectable formulations of sumatriptan. Both of these are more costly than oral sumatriptan, and injectable sumatriptan has more side effects than oral triptans.

Combining treatment with metoclopramide can be helpful. In a study by Schulman and Dermott looking at patients who had previously been triptan nonresponders, 63% of those who took metoclopramide with sumatriptan had meaningful pain relief, compared with 31% of those who received sumatriptan and placebo.2

In a different study, Tfelt-Hansen et al. compared treatment with the combination of lysine acetylsalicylate plus metoclopramide versus treatment with 100 mg of sumatriptan.3 There was no difference in outcomes between the two treatment groups, with the lysine acetylsalicylate plus metoclopramide patients having a 57% success rate for first treated migraine compared with 53% of the sumatriptan-treated patients.

Dr. Douglas S. Paauw, University of Washington, Seattle
Dr. Douglas S. Paauw

Treating with the combination of naproxen plus sumatriptan is superior to treating with either medication alone. Brandes et al. reported on two studies involving the use of the sumatriptan/naproxen combination, compared with using sumatriptan, naproxen, or placebo.4 In both, taking the sumatriptan/naproxen combination was superior to taking sumatriptan, naproxen, or placebo (P < .001).

In a study of patients with poor prior response to triptans, Mathew et al. found that the sumatriptan/naproxen combination was superior to placebo for both 2- and 24-hour headache relief (P < .001).5
 

Pearl

Try several options before abandoning triptans in previous triptan nonresponders, including trying a different triptan, adding metoclopramide, orcombining with an NSAID.

Dr. Paauw is professor of medicine in the division of general internal medicine at the University of Washington, Seattle, and he serves as third-year medical student clerkship director at the University of Washington. He is a member of the editorial advisory board of Internal Medicine News. Dr. Paauw has no conflicts to disclose. Contact him at imnews@mdedge.com.

References

1. Dahlöf CG. Infrequent or nonresponse to oral sumatriptan does not predict response to other triptans – review of four trials. Cephalalgia. 2006 Feb;26(2):98-106.

2. Schulman EA, Dermott KF. Sumatriptan plus metoclopramide in triptan-nonresponsive migraineurs. Headache. 2003 Jul-Aug;43(7):729-33.

3. Tfelt-Hansen P et al. The effectiveness of combined oral lysine acetylsalicylate and metoclopramide compared with oral sumatriptan for migraine. Lancet. 1995 Oct 7;346(8980):923-6.

4. Brandes JL et al. Sumatriptan‐naproxen for acute treatment of migraine: A randomized trial. JAMA. 2007;297:1443‐54.

5. Mathew NT, Landy S, Stark S, et al. Fixed‐dose sumatriptan and naproxen in poor responders to triptans with a short half‐life. Headache. 2009;49:971‐82.

A 25-year-old woman presents to discuss treatment of her headaches. They occur two or three times a month and last for 4-6 hours. The headaches are disabling, have a pounding quality behind the patient’s right eye, and worsen with exercise. The patient’s neurologic exam is normal.

She has tried oral sumatriptan and naproxen, but neither drug provided her with any relief from the headaches. What treatment would you recommend?
 

A. Topiramate

B. Beta-blocker

C. Lasmiditan

D. Metoclopramide plus sumatriptan

E. Ubrogepant

It is common to see patients with migraine headaches and to see patients with migraines who have not responded to previous migraine therapies. This column will focus on some migraine therapy pearls.

DKart/iStockphoto

For this patient, I would try choice D first, giving metoclopramide with oral sumatriptan to see if it can improve response to sumatriptan. The two new classes of drugs for acute migraine therapy, the gepants and ditans, certainly have a role in patients unresponsive or intolerant of triptans/NSAIDS, but I would try several tricks with these less expensive medications first before entering into prior authorization hell trying to get a gepant or ditan.

When a patient has already used a triptan but experienced no benefit from it, often the next medication a patient tries is a different triptan. Dahlof reviewed four trials that looked at the efficacy of switching sumatriptan nonresponders to a different triptan and found that lack of response to sumatriptan did not predict lack of response to an alternative triptan.1 Unfortunately, acquiring insurance coverage for an alternate triptan can be difficult.

Other treatment options are nasal or injectable formulations of sumatriptan. Both of these are more costly than oral sumatriptan, and injectable sumatriptan has more side effects than oral triptans.

Combining treatment with metoclopramide can be helpful. In a study by Schulman and Dermott looking at patients who had previously been triptan nonresponders, 63% of those who took metoclopramide with sumatriptan had meaningful pain relief, compared with 31% of those who received sumatriptan and placebo.2

In a different study, Tfelt-Hansen et al. compared treatment with the combination of lysine acetylsalicylate plus metoclopramide versus treatment with 100 mg of sumatriptan.3 There was no difference in outcomes between the two treatment groups, with the lysine acetylsalicylate plus metoclopramide patients having a 57% success rate for first treated migraine compared with 53% of the sumatriptan-treated patients.

Dr. Douglas S. Paauw, University of Washington, Seattle
Dr. Douglas S. Paauw

Treating with the combination of naproxen plus sumatriptan is superior to treating with either medication alone. Brandes et al. reported on two studies involving the use of the sumatriptan/naproxen combination, compared with using sumatriptan, naproxen, or placebo.4 In both, taking the sumatriptan/naproxen combination was superior to taking sumatriptan, naproxen, or placebo (P < .001).

In a study of patients with poor prior response to triptans, Mathew et al. found that the sumatriptan/naproxen combination was superior to placebo for both 2- and 24-hour headache relief (P < .001).5
 

Pearl

Try several options before abandoning triptans in previous triptan nonresponders, including trying a different triptan, adding metoclopramide, orcombining with an NSAID.

Dr. Paauw is professor of medicine in the division of general internal medicine at the University of Washington, Seattle, and he serves as third-year medical student clerkship director at the University of Washington. He is a member of the editorial advisory board of Internal Medicine News. Dr. Paauw has no conflicts to disclose. Contact him at imnews@mdedge.com.

References

1. Dahlöf CG. Infrequent or nonresponse to oral sumatriptan does not predict response to other triptans – review of four trials. Cephalalgia. 2006 Feb;26(2):98-106.

2. Schulman EA, Dermott KF. Sumatriptan plus metoclopramide in triptan-nonresponsive migraineurs. Headache. 2003 Jul-Aug;43(7):729-33.

3. Tfelt-Hansen P et al. The effectiveness of combined oral lysine acetylsalicylate and metoclopramide compared with oral sumatriptan for migraine. Lancet. 1995 Oct 7;346(8980):923-6.

4. Brandes JL et al. Sumatriptan‐naproxen for acute treatment of migraine: A randomized trial. JAMA. 2007;297:1443‐54.

5. Mathew NT, Landy S, Stark S, et al. Fixed‐dose sumatriptan and naproxen in poor responders to triptans with a short half‐life. Headache. 2009;49:971‐82.

Publications
Publications
Topics
Article Type
Sections
Disallow All Ads
Content Gating
No Gating (article Unlocked/Free)
Alternative CME
Disqus Comments
Default
Use ProPublica
Hide sidebar & use full width
render the right sidebar.
Conference Recap Checkbox
Not Conference Recap
Clinical Edge
Display the Slideshow in this Article

Can this patient get IV contrast?

Article Type
Changed
Mon, 03/02/2020 - 21:52

A 59-year-old man is admitted with abdominal pain. He has a history of pancreatitis. A contrast CT scan is ordered. He reports a history of severe shellfish allergy when the radiology tech checks him in for the procedure. You are paged regarding what to do:

A) Continue with scan as ordered.

B) Switch to MRI scan.

C) Switch to MRI scan with gadolinium.

D) Continue with CT with contrast, give dose of Solu-Medrol.

E) Continue with CT with contrast give IV diphenhydramine.
 

The correct answer here is A, This patient can receive his scan and receive contrast as ordered.

For many years, patients have been asked about shellfish allergy as a proxy for having increased risk when receiving iodine containing contrast. The mistaken thought was that shellfish contains iodine, so allergy to shellfish was likely to portend allergy to iodine.

Dr. Douglas S. Paauw, University of Washington, Seattle
Dr. Douglas S. Paauw

Allergy to shellfish is caused by individual proteins that are definitely not in iodine-containing contrast.1 Beaty et al. studied the prevalence of the belief that allergy to shellfish is tied to iodine allergy in a survey given to 231 faculty radiologists and interventional cardiologists.2 Almost 70% responded that they inquire about seafood allergy before procedures that require iodine contrast, and 37% reported they would withhold the contrast or premedicate patients if they had a seafood allergy.

In a more recent study, Westermann-Clark and colleagues surveyed 252 health professionals before and after an educational intervention to dispel the myth of shellfish allergy and iodinated contrast reactions.3 Before the intervention, 66% of participants felt it was important to ask about shellfish allergies and 93% felt it was important to ask about iodine allergies; 26% responded that they would withhold iodinated contrast material in patients with a shellfish allergy, and 56% would withhold in patients with an iodine allergy. A total of 62% reported they would premedicate patients with a shellfish allergy and 75% would premedicate patients with an iodine allergy. The numbers declined dramatically after the educational intervention.

Patients who have seafood allergy have a higher rate of reactions to iodinated contrast, but not at a higher rate than do patients with other food allergies or asthma.4 Most radiology departments do not screen for other food allergies despite the fact these allergies have the same increased risk as for patients with a seafood/shellfish allergy. These patients are more allergic, and in general, are more likely to have reactions. The American Academy of Allergy, Asthma, and Immunology recommends not routinely ordering low- or iso-osmolar radiocontrast media or pretreating with either antihistamines or steroids in patients with a history of seafood allergy.5



There is no evidence that iodine causes allergic reactions. It makes sense that iodine does not cause allergic reactions, as it is an essential component in the human body, in thyroid hormone and in amino acids.6 Patients with dermatitis following topical application of iodine preparations such as povidone-iodide are not reacting to the iodine.

Van Ketel and van den Berg patch-tested patients with a history of dermatitis after exposure to povidone-iodine.7 All patients reacted to patch testing with povidone-iodine, but none reacted to direct testing to iodine (0/5 with patch testing of potassium iodide and 0/3 with testing with iodine tincture).


Take home points:

  • It is unnecessary and unhelpful to ask patients about seafood allergies before ordering radiologic studies involving contrast.
  • Iodine allergy does not exist.

Dr. Paauw is professor of medicine in the division of general internal medicine at the University of Washington, Seattle, and he serves as third-year medical student clerkship director at the University of Washington. Contact Dr. Paauw at dpaauw@uw.edu.

References

1. Narayan AK et al. Avoiding contrast-enhanced computed tomography scans in patients with shellfish allergies. J Hosp Med. 2016 Jun;11(6):435-7.

2. Beaty AD et al. Seafood allergy and radiocontrast media: Are physicians propagating a myth? Am J Med. 2008 Feb;121(2):158.e1-4.

3. Westermann-Clark E et al. Debunking myths about “allergy” to radiocontrast media in an academic institution. Postgrad Med. 2015 Apr;127(3):295-300.

4. Coakley FV and DM Panicek. Iodine allergy: An oyster without a pearl? AJR Am J Roentgenol. 1997 Oct;169(4):951-2.

5. American Academy of Allergy, Asthma & Immunology recommendations on low- or iso-osmolar radiocontrast media.

6. Schabelman E and M Witting. The relationship of radiocontrast, iodine, and seafood allergies: A medical myth exposed. J Emerg Med. 2010 Nov;39(5):701-7.

7. van Ketel WG and WH van den Berg. Sensitization to povidone-iodine. Dermatol Clin. 1990 Jan;8(1):107-9.

Publications
Topics
Sections

A 59-year-old man is admitted with abdominal pain. He has a history of pancreatitis. A contrast CT scan is ordered. He reports a history of severe shellfish allergy when the radiology tech checks him in for the procedure. You are paged regarding what to do:

A) Continue with scan as ordered.

B) Switch to MRI scan.

C) Switch to MRI scan with gadolinium.

D) Continue with CT with contrast, give dose of Solu-Medrol.

E) Continue with CT with contrast give IV diphenhydramine.
 

The correct answer here is A, This patient can receive his scan and receive contrast as ordered.

For many years, patients have been asked about shellfish allergy as a proxy for having increased risk when receiving iodine containing contrast. The mistaken thought was that shellfish contains iodine, so allergy to shellfish was likely to portend allergy to iodine.

Dr. Douglas S. Paauw, University of Washington, Seattle
Dr. Douglas S. Paauw

Allergy to shellfish is caused by individual proteins that are definitely not in iodine-containing contrast.1 Beaty et al. studied the prevalence of the belief that allergy to shellfish is tied to iodine allergy in a survey given to 231 faculty radiologists and interventional cardiologists.2 Almost 70% responded that they inquire about seafood allergy before procedures that require iodine contrast, and 37% reported they would withhold the contrast or premedicate patients if they had a seafood allergy.

In a more recent study, Westermann-Clark and colleagues surveyed 252 health professionals before and after an educational intervention to dispel the myth of shellfish allergy and iodinated contrast reactions.3 Before the intervention, 66% of participants felt it was important to ask about shellfish allergies and 93% felt it was important to ask about iodine allergies; 26% responded that they would withhold iodinated contrast material in patients with a shellfish allergy, and 56% would withhold in patients with an iodine allergy. A total of 62% reported they would premedicate patients with a shellfish allergy and 75% would premedicate patients with an iodine allergy. The numbers declined dramatically after the educational intervention.

Patients who have seafood allergy have a higher rate of reactions to iodinated contrast, but not at a higher rate than do patients with other food allergies or asthma.4 Most radiology departments do not screen for other food allergies despite the fact these allergies have the same increased risk as for patients with a seafood/shellfish allergy. These patients are more allergic, and in general, are more likely to have reactions. The American Academy of Allergy, Asthma, and Immunology recommends not routinely ordering low- or iso-osmolar radiocontrast media or pretreating with either antihistamines or steroids in patients with a history of seafood allergy.5



There is no evidence that iodine causes allergic reactions. It makes sense that iodine does not cause allergic reactions, as it is an essential component in the human body, in thyroid hormone and in amino acids.6 Patients with dermatitis following topical application of iodine preparations such as povidone-iodide are not reacting to the iodine.

Van Ketel and van den Berg patch-tested patients with a history of dermatitis after exposure to povidone-iodine.7 All patients reacted to patch testing with povidone-iodine, but none reacted to direct testing to iodine (0/5 with patch testing of potassium iodide and 0/3 with testing with iodine tincture).


Take home points:

  • It is unnecessary and unhelpful to ask patients about seafood allergies before ordering radiologic studies involving contrast.
  • Iodine allergy does not exist.

Dr. Paauw is professor of medicine in the division of general internal medicine at the University of Washington, Seattle, and he serves as third-year medical student clerkship director at the University of Washington. Contact Dr. Paauw at dpaauw@uw.edu.

References

1. Narayan AK et al. Avoiding contrast-enhanced computed tomography scans in patients with shellfish allergies. J Hosp Med. 2016 Jun;11(6):435-7.

2. Beaty AD et al. Seafood allergy and radiocontrast media: Are physicians propagating a myth? Am J Med. 2008 Feb;121(2):158.e1-4.

3. Westermann-Clark E et al. Debunking myths about “allergy” to radiocontrast media in an academic institution. Postgrad Med. 2015 Apr;127(3):295-300.

4. Coakley FV and DM Panicek. Iodine allergy: An oyster without a pearl? AJR Am J Roentgenol. 1997 Oct;169(4):951-2.

5. American Academy of Allergy, Asthma & Immunology recommendations on low- or iso-osmolar radiocontrast media.

6. Schabelman E and M Witting. The relationship of radiocontrast, iodine, and seafood allergies: A medical myth exposed. J Emerg Med. 2010 Nov;39(5):701-7.

7. van Ketel WG and WH van den Berg. Sensitization to povidone-iodine. Dermatol Clin. 1990 Jan;8(1):107-9.

A 59-year-old man is admitted with abdominal pain. He has a history of pancreatitis. A contrast CT scan is ordered. He reports a history of severe shellfish allergy when the radiology tech checks him in for the procedure. You are paged regarding what to do:

A) Continue with scan as ordered.

B) Switch to MRI scan.

C) Switch to MRI scan with gadolinium.

D) Continue with CT with contrast, give dose of Solu-Medrol.

E) Continue with CT with contrast give IV diphenhydramine.
 

The correct answer here is A, This patient can receive his scan and receive contrast as ordered.

For many years, patients have been asked about shellfish allergy as a proxy for having increased risk when receiving iodine containing contrast. The mistaken thought was that shellfish contains iodine, so allergy to shellfish was likely to portend allergy to iodine.

Dr. Douglas S. Paauw, University of Washington, Seattle
Dr. Douglas S. Paauw

Allergy to shellfish is caused by individual proteins that are definitely not in iodine-containing contrast.1 Beaty et al. studied the prevalence of the belief that allergy to shellfish is tied to iodine allergy in a survey given to 231 faculty radiologists and interventional cardiologists.2 Almost 70% responded that they inquire about seafood allergy before procedures that require iodine contrast, and 37% reported they would withhold the contrast or premedicate patients if they had a seafood allergy.

In a more recent study, Westermann-Clark and colleagues surveyed 252 health professionals before and after an educational intervention to dispel the myth of shellfish allergy and iodinated contrast reactions.3 Before the intervention, 66% of participants felt it was important to ask about shellfish allergies and 93% felt it was important to ask about iodine allergies; 26% responded that they would withhold iodinated contrast material in patients with a shellfish allergy, and 56% would withhold in patients with an iodine allergy. A total of 62% reported they would premedicate patients with a shellfish allergy and 75% would premedicate patients with an iodine allergy. The numbers declined dramatically after the educational intervention.

Patients who have seafood allergy have a higher rate of reactions to iodinated contrast, but not at a higher rate than do patients with other food allergies or asthma.4 Most radiology departments do not screen for other food allergies despite the fact these allergies have the same increased risk as for patients with a seafood/shellfish allergy. These patients are more allergic, and in general, are more likely to have reactions. The American Academy of Allergy, Asthma, and Immunology recommends not routinely ordering low- or iso-osmolar radiocontrast media or pretreating with either antihistamines or steroids in patients with a history of seafood allergy.5



There is no evidence that iodine causes allergic reactions. It makes sense that iodine does not cause allergic reactions, as it is an essential component in the human body, in thyroid hormone and in amino acids.6 Patients with dermatitis following topical application of iodine preparations such as povidone-iodide are not reacting to the iodine.

Van Ketel and van den Berg patch-tested patients with a history of dermatitis after exposure to povidone-iodine.7 All patients reacted to patch testing with povidone-iodine, but none reacted to direct testing to iodine (0/5 with patch testing of potassium iodide and 0/3 with testing with iodine tincture).


Take home points:

  • It is unnecessary and unhelpful to ask patients about seafood allergies before ordering radiologic studies involving contrast.
  • Iodine allergy does not exist.

Dr. Paauw is professor of medicine in the division of general internal medicine at the University of Washington, Seattle, and he serves as third-year medical student clerkship director at the University of Washington. Contact Dr. Paauw at dpaauw@uw.edu.

References

1. Narayan AK et al. Avoiding contrast-enhanced computed tomography scans in patients with shellfish allergies. J Hosp Med. 2016 Jun;11(6):435-7.

2. Beaty AD et al. Seafood allergy and radiocontrast media: Are physicians propagating a myth? Am J Med. 2008 Feb;121(2):158.e1-4.

3. Westermann-Clark E et al. Debunking myths about “allergy” to radiocontrast media in an academic institution. Postgrad Med. 2015 Apr;127(3):295-300.

4. Coakley FV and DM Panicek. Iodine allergy: An oyster without a pearl? AJR Am J Roentgenol. 1997 Oct;169(4):951-2.

5. American Academy of Allergy, Asthma & Immunology recommendations on low- or iso-osmolar radiocontrast media.

6. Schabelman E and M Witting. The relationship of radiocontrast, iodine, and seafood allergies: A medical myth exposed. J Emerg Med. 2010 Nov;39(5):701-7.

7. van Ketel WG and WH van den Berg. Sensitization to povidone-iodine. Dermatol Clin. 1990 Jan;8(1):107-9.

Publications
Publications
Topics
Article Type
Sections
Disallow All Ads
Content Gating
No Gating (article Unlocked/Free)
Alternative CME
Disqus Comments
Default
Use ProPublica
Hide sidebar & use full width
render the right sidebar.

Common drug with lots of surprising side effects

Article Type
Changed
Wed, 01/22/2020 - 12:45

A 55-year-old woman comes to clinic for follow-up. She reports her family is worried that she isn’t getting enough sleep and is more tired than usual. The patient reports she is sleeping 8 hours a night and wakes up feeling rested, but she has noticed she has been yawning much more frequently than she remembers in the past.

Dr. Douglas S. Paauw, University of Washington, Seattle
Dr. Douglas S. Paauw

Past medical history: gastroesophageal reflux disease, hypertension, generalized anxiety disorder, hypothyroidism, and osteoporosis. Medications: amlodipine, lansoprazole, irbesartan, escitalopram, levothyroxine, and alendronate. Physical examination: blood pressure 110/70 mm Hg, pulse 60 bpm. Lower extremities: 1+ edema.

What is the likely cause of her increased yawning?

A. Amlodipine.

B. Alendronate.

C. Irbesartan.

D. Escitalopram.

E. Lansoprazole.

The correct answer here is escitalopram. Selective serotonin reuptake inhibitors in general are well tolerated. Given how commonly these drugs are used, however, there are a number of lesser-known side effects that you are likely to see.

In the above case, this patient has yawning caused by her SSRI. Roncero et al. described a case of yawning in a patient on escitalopram that resolved when the dose of escitalopram was reduced.1 Paroxetine has been reported to cause yawning at both low and high doses.2

In a review of drug-induced yawning, SSRIs as a class were most frequently involved, and sertraline and fluoxetine were implicated in addition to paroxetine.3 The serotonin-norepinephrine reuptake inhibitors duloxetine and venlafaxine have also been associated with yawning.4,5

Hyperhydrosis has also been linked to SSRIs and SNRIs, and both yawning and hyperhidrosis may occur because of an underlying thermoregulatory dysfunction.6

SSRIs have been linked to increased bleeding risk, especially increased risk of upper gastrointestinal hemorrhage. Laporte and colleagues showed an association of SSRI use and risk of bleeding in a meta-analysis of 42 observational studies, with an odds ratio of 1.41 (95% confidence interval, 1.27-1.57; P less than .0001).7 The risk of upper gastrointestinal (UGI) bleeding is further increased if patients are also taking NSAIDs.

Anglin et al. looked at 15 case-control studies and 4 cohort studies and found an OR of 1.66 for UGI bleeding with SSRI use, and an OR of 4.25 for UGI bleeding if SSRI use was combined with NSAID use.8 The number needed to harm is 3,177 for NSAID use in populations at low risk for GI bleeding, but it is much lower (881) in higher-risk populations.8 Make sure to think about patients’ bleeding risks when starting SSRIs.

An issue that comes up frequently is: What is the risk of bleeding in patients on SSRIs who are also on anticoagulants? Dr. Quinn and colleagues looked at the bleeding risk of anticoagulated patients also taking SSRIs in the ROCKET AF trial.9 They found 737 patients who received SSRIs and matched them with other patients not on SSRIs in the trial. All patients in the trial were either receiving rivaroxaban or warfarin for stroke prophylaxis. They found no significant increase risk in bleeding in the patients on SSRIs and anticoagulants.
 

Take-home points:

  • Yawning and hyperhidrosis are interesting side effects of SSRIs.
  • Bleeding risk is increased in patients on SSRIs, especially when combined with NSAIDs.

Dr. Paauw is professor of medicine in the division of general internal medicine at the University of Washington, Seattle, and he serves as third-year medical student clerkship director at the University of Washington. Contact Dr. Paauw at dpaauw@uw.edu.

References

1. Neurologia. 2013 Nov-Dec;28(9):589-90.

2. Psychiatry Clin Neurosci. 2006 Apr;60(2):260.

3. Presse Med. 2014 Oct;43(10 Pt 1):1135-6.

4. Prog Neuropsychopharmacol Biol Psychiatry. 2009 Jun 15;33(4):747.

5. Ann Pharmacother. 2011 Oct;45(10):1297-301.

6. Depress Anxiety. 2017 Dec;34(12):1134-46.

7. Pharmacol Res. 2017 Apr;118:19-32.

8. Am J Gastroenterol. 2014 Jun;109(6):811-9.

9. J Am Heart Assoc. 2018 Aug 7;7(15):e008755.

Publications
Topics
Sections

A 55-year-old woman comes to clinic for follow-up. She reports her family is worried that she isn’t getting enough sleep and is more tired than usual. The patient reports she is sleeping 8 hours a night and wakes up feeling rested, but she has noticed she has been yawning much more frequently than she remembers in the past.

Dr. Douglas S. Paauw, University of Washington, Seattle
Dr. Douglas S. Paauw

Past medical history: gastroesophageal reflux disease, hypertension, generalized anxiety disorder, hypothyroidism, and osteoporosis. Medications: amlodipine, lansoprazole, irbesartan, escitalopram, levothyroxine, and alendronate. Physical examination: blood pressure 110/70 mm Hg, pulse 60 bpm. Lower extremities: 1+ edema.

What is the likely cause of her increased yawning?

A. Amlodipine.

B. Alendronate.

C. Irbesartan.

D. Escitalopram.

E. Lansoprazole.

The correct answer here is escitalopram. Selective serotonin reuptake inhibitors in general are well tolerated. Given how commonly these drugs are used, however, there are a number of lesser-known side effects that you are likely to see.

In the above case, this patient has yawning caused by her SSRI. Roncero et al. described a case of yawning in a patient on escitalopram that resolved when the dose of escitalopram was reduced.1 Paroxetine has been reported to cause yawning at both low and high doses.2

In a review of drug-induced yawning, SSRIs as a class were most frequently involved, and sertraline and fluoxetine were implicated in addition to paroxetine.3 The serotonin-norepinephrine reuptake inhibitors duloxetine and venlafaxine have also been associated with yawning.4,5

Hyperhydrosis has also been linked to SSRIs and SNRIs, and both yawning and hyperhidrosis may occur because of an underlying thermoregulatory dysfunction.6

SSRIs have been linked to increased bleeding risk, especially increased risk of upper gastrointestinal hemorrhage. Laporte and colleagues showed an association of SSRI use and risk of bleeding in a meta-analysis of 42 observational studies, with an odds ratio of 1.41 (95% confidence interval, 1.27-1.57; P less than .0001).7 The risk of upper gastrointestinal (UGI) bleeding is further increased if patients are also taking NSAIDs.

Anglin et al. looked at 15 case-control studies and 4 cohort studies and found an OR of 1.66 for UGI bleeding with SSRI use, and an OR of 4.25 for UGI bleeding if SSRI use was combined with NSAID use.8 The number needed to harm is 3,177 for NSAID use in populations at low risk for GI bleeding, but it is much lower (881) in higher-risk populations.8 Make sure to think about patients’ bleeding risks when starting SSRIs.

An issue that comes up frequently is: What is the risk of bleeding in patients on SSRIs who are also on anticoagulants? Dr. Quinn and colleagues looked at the bleeding risk of anticoagulated patients also taking SSRIs in the ROCKET AF trial.9 They found 737 patients who received SSRIs and matched them with other patients not on SSRIs in the trial. All patients in the trial were either receiving rivaroxaban or warfarin for stroke prophylaxis. They found no significant increase risk in bleeding in the patients on SSRIs and anticoagulants.
 

Take-home points:

  • Yawning and hyperhidrosis are interesting side effects of SSRIs.
  • Bleeding risk is increased in patients on SSRIs, especially when combined with NSAIDs.

Dr. Paauw is professor of medicine in the division of general internal medicine at the University of Washington, Seattle, and he serves as third-year medical student clerkship director at the University of Washington. Contact Dr. Paauw at dpaauw@uw.edu.

References

1. Neurologia. 2013 Nov-Dec;28(9):589-90.

2. Psychiatry Clin Neurosci. 2006 Apr;60(2):260.

3. Presse Med. 2014 Oct;43(10 Pt 1):1135-6.

4. Prog Neuropsychopharmacol Biol Psychiatry. 2009 Jun 15;33(4):747.

5. Ann Pharmacother. 2011 Oct;45(10):1297-301.

6. Depress Anxiety. 2017 Dec;34(12):1134-46.

7. Pharmacol Res. 2017 Apr;118:19-32.

8. Am J Gastroenterol. 2014 Jun;109(6):811-9.

9. J Am Heart Assoc. 2018 Aug 7;7(15):e008755.

A 55-year-old woman comes to clinic for follow-up. She reports her family is worried that she isn’t getting enough sleep and is more tired than usual. The patient reports she is sleeping 8 hours a night and wakes up feeling rested, but she has noticed she has been yawning much more frequently than she remembers in the past.

Dr. Douglas S. Paauw, University of Washington, Seattle
Dr. Douglas S. Paauw

Past medical history: gastroesophageal reflux disease, hypertension, generalized anxiety disorder, hypothyroidism, and osteoporosis. Medications: amlodipine, lansoprazole, irbesartan, escitalopram, levothyroxine, and alendronate. Physical examination: blood pressure 110/70 mm Hg, pulse 60 bpm. Lower extremities: 1+ edema.

What is the likely cause of her increased yawning?

A. Amlodipine.

B. Alendronate.

C. Irbesartan.

D. Escitalopram.

E. Lansoprazole.

The correct answer here is escitalopram. Selective serotonin reuptake inhibitors in general are well tolerated. Given how commonly these drugs are used, however, there are a number of lesser-known side effects that you are likely to see.

In the above case, this patient has yawning caused by her SSRI. Roncero et al. described a case of yawning in a patient on escitalopram that resolved when the dose of escitalopram was reduced.1 Paroxetine has been reported to cause yawning at both low and high doses.2

In a review of drug-induced yawning, SSRIs as a class were most frequently involved, and sertraline and fluoxetine were implicated in addition to paroxetine.3 The serotonin-norepinephrine reuptake inhibitors duloxetine and venlafaxine have also been associated with yawning.4,5

Hyperhydrosis has also been linked to SSRIs and SNRIs, and both yawning and hyperhidrosis may occur because of an underlying thermoregulatory dysfunction.6

SSRIs have been linked to increased bleeding risk, especially increased risk of upper gastrointestinal hemorrhage. Laporte and colleagues showed an association of SSRI use and risk of bleeding in a meta-analysis of 42 observational studies, with an odds ratio of 1.41 (95% confidence interval, 1.27-1.57; P less than .0001).7 The risk of upper gastrointestinal (UGI) bleeding is further increased if patients are also taking NSAIDs.

Anglin et al. looked at 15 case-control studies and 4 cohort studies and found an OR of 1.66 for UGI bleeding with SSRI use, and an OR of 4.25 for UGI bleeding if SSRI use was combined with NSAID use.8 The number needed to harm is 3,177 for NSAID use in populations at low risk for GI bleeding, but it is much lower (881) in higher-risk populations.8 Make sure to think about patients’ bleeding risks when starting SSRIs.

An issue that comes up frequently is: What is the risk of bleeding in patients on SSRIs who are also on anticoagulants? Dr. Quinn and colleagues looked at the bleeding risk of anticoagulated patients also taking SSRIs in the ROCKET AF trial.9 They found 737 patients who received SSRIs and matched them with other patients not on SSRIs in the trial. All patients in the trial were either receiving rivaroxaban or warfarin for stroke prophylaxis. They found no significant increase risk in bleeding in the patients on SSRIs and anticoagulants.
 

Take-home points:

  • Yawning and hyperhidrosis are interesting side effects of SSRIs.
  • Bleeding risk is increased in patients on SSRIs, especially when combined with NSAIDs.

Dr. Paauw is professor of medicine in the division of general internal medicine at the University of Washington, Seattle, and he serves as third-year medical student clerkship director at the University of Washington. Contact Dr. Paauw at dpaauw@uw.edu.

References

1. Neurologia. 2013 Nov-Dec;28(9):589-90.

2. Psychiatry Clin Neurosci. 2006 Apr;60(2):260.

3. Presse Med. 2014 Oct;43(10 Pt 1):1135-6.

4. Prog Neuropsychopharmacol Biol Psychiatry. 2009 Jun 15;33(4):747.

5. Ann Pharmacother. 2011 Oct;45(10):1297-301.

6. Depress Anxiety. 2017 Dec;34(12):1134-46.

7. Pharmacol Res. 2017 Apr;118:19-32.

8. Am J Gastroenterol. 2014 Jun;109(6):811-9.

9. J Am Heart Assoc. 2018 Aug 7;7(15):e008755.

Publications
Publications
Topics
Article Type
Sections
Disallow All Ads
Content Gating
No Gating (article Unlocked/Free)
Alternative CME
Disqus Comments
Default
Use ProPublica
Hide sidebar & use full width
render the right sidebar.

My favorite natural treatments

Article Type
Changed
Thu, 12/19/2019 - 11:28

I practice in Seattle, where many of my patients are interested in natural treatment options. I am always puzzled that so many people assume that natural treatment options would be safer than prescription medications. Tsunamis, wildfires, flooding, and earthquakes are all natural and are deadly. There are certainly many natural poisons. There are a number of natural treatments that are very helpful, and recommending them are an important part of my practice. I want to share a few with you.

A useful vein pursuit?

Dr. Douglas S. Paauw, University of Washington, Seattle
Dr. Douglas S. Paauw

Case: A 60-year-old woman presents to clinic with increasing pain in her left leg. She had a deep vein thrombosis in her left leg 2 years ago, which involved a large portion of her superficial femoral vein. She has noticed edema over the past few weeks, and pain has been more severe over the past 3 months. On exam, varicosities on left lower extremity with grade 2+ edema. Duplex of the lower extremity does not show deep vein thrombosis, but does show a great deal of venous valvular incompetence. She has tried compression stockings for the past 2 weeks. What is the best treatment option?

A) Turmeric

B) Amitriptyline

C) Vitamin B12

D) Horse chestnut

The treatment option with positive data for this problem is horse chestnut. What is horse chestnut? Horse chestnut is a kind of tree, and the seed extract contains aescin which is believed to be the active ingredient. Diehm et al. studied horse chestnut seed extract (HCSE), compared with compression stockings and placebo for edema from chronic venous insufficiency in a study of 240 patients.1 Lower-leg volume decreased by 43 mL with HCSE, 46 mL with compression stockings, and increased by 9 mL with placebo (P less than .005 for HCSE and P less than .002 for compression). In a Cochrane review, HCSE was considered efficacious and safe for the short-term therapy for chronic venous insufficiency.2 Studies have shown both an improvement in pain as well as swelling in patients with chronic venous insufficiency.

The question of UTIs

Probably the most popular natural treatment for prevention and treatment of urinary tract infections (UTIs) in women is cranberry juice (or cranberry extract). Unfortunately, there is little evidence that this treatment is helpful. In a Cochrane analysis, the conclusion was, based on current evidence, cranberry juice cannot currently be recommended for the prevention of UTIs.3 A natural product that appears to be more promising is the sugar D-mannose. Kranjčec et al. studied 308 women with acute UTI who had a history of recurrent UTI.4 All the women were treated for their symptomatic infection with ciprofloxacin (500 mg twice daily for 1 week). The women were allocated equally to three groups for 6 months: D-mannose 2 g daily, nitrofurantoin 50 mg daily, or no prophylaxis. About 60% of the women who received no prophylaxis had a UTI during the study period, compared with 20% in the nitrofurantoin group and 15% in the D-mannose group. The relative risk for D-mannose, compared with no prophylaxis, was 0.24 and for nitrofurantoin was 0.34 (P less than .0001), compared with no prophylaxis.

 

 

Made for migraines?

Migraine prophylaxis is challenging because all medications that are commonly used have side effects that often limit patient adherence. Tricyclic antidepressants (dry mouth, dizziness and weight gain), beta-blockers (fatigue, decreased exercise tolerance), valproate (weight gain and fatigue), and topiramate (parasthesias and mental slowing) all have troubling side effects. Riboflavin is a vitamin with evidence of effectiveness for migraine prophylaxis. It is extremely well tolerated. In a recent study in children with migraines, Talebian et al. studied 90 children with migraines who were randomized to three groups (200 mg of riboflavin a day, 100 mg of riboflavin a day, or placebo) after observation during a 1-month baseline period.5 There was a significant reduction in migraine frequency and duration in patients receiving 200 mg of riboflavin daily, compared with placebo. Rahimdel et al. published an interesting study comparing high-dose riboflavin with valproate for migraine prophylaxis. A total of 90 patients were randomized to receive 400 mg of riboflavin or 500 mg of valproate over a 12-month study.6 Both treatments resulted in marked reduction in frequency, duration, and severity of migraines (not statistically significantly different from each other). The reduction in migraine frequency for the riboflavin group was from 9.2 headache days per month to 2.4. The American Academy of Neurology rates the level of evidence for riboflavin as B.

Pearl

Horse chestnut, D-mannose, and riboflavin are safe alternative therapies that can be helpful for several common problems we see frequently in primary care.

References

1. Diehm C et al. Lancet. 1996;347(8997):292-4.

2. Pittler MH, Ernst E. Cochrane Database Syst Rev. 2012 Nov 14;11:CD003230.

3. Jepson RG et al. Cochrane Database Syst Rev. 2012;10:CD001321.

4. Kranjčec B et al. World J Urol. 2014 Feb;32(1):79-84.

5. Talebian A et al. Electron Physician. Feb 25;10(2):6279-85.

6. Rahimdel A et al. Electron Physician. 2015 Oct 19;7(6):1344-8.

Dr. Paauw is professor of medicine in the division of general internal medicine at the University of Washington, Seattle, and he serves as third-year medical student clerkship director at the University of Washington. Contact Dr. Paauw at dpaauw@uw.edu.

Publications
Topics
Sections

I practice in Seattle, where many of my patients are interested in natural treatment options. I am always puzzled that so many people assume that natural treatment options would be safer than prescription medications. Tsunamis, wildfires, flooding, and earthquakes are all natural and are deadly. There are certainly many natural poisons. There are a number of natural treatments that are very helpful, and recommending them are an important part of my practice. I want to share a few with you.

A useful vein pursuit?

Dr. Douglas S. Paauw, University of Washington, Seattle
Dr. Douglas S. Paauw

Case: A 60-year-old woman presents to clinic with increasing pain in her left leg. She had a deep vein thrombosis in her left leg 2 years ago, which involved a large portion of her superficial femoral vein. She has noticed edema over the past few weeks, and pain has been more severe over the past 3 months. On exam, varicosities on left lower extremity with grade 2+ edema. Duplex of the lower extremity does not show deep vein thrombosis, but does show a great deal of venous valvular incompetence. She has tried compression stockings for the past 2 weeks. What is the best treatment option?

A) Turmeric

B) Amitriptyline

C) Vitamin B12

D) Horse chestnut

The treatment option with positive data for this problem is horse chestnut. What is horse chestnut? Horse chestnut is a kind of tree, and the seed extract contains aescin which is believed to be the active ingredient. Diehm et al. studied horse chestnut seed extract (HCSE), compared with compression stockings and placebo for edema from chronic venous insufficiency in a study of 240 patients.1 Lower-leg volume decreased by 43 mL with HCSE, 46 mL with compression stockings, and increased by 9 mL with placebo (P less than .005 for HCSE and P less than .002 for compression). In a Cochrane review, HCSE was considered efficacious and safe for the short-term therapy for chronic venous insufficiency.2 Studies have shown both an improvement in pain as well as swelling in patients with chronic venous insufficiency.

The question of UTIs

Probably the most popular natural treatment for prevention and treatment of urinary tract infections (UTIs) in women is cranberry juice (or cranberry extract). Unfortunately, there is little evidence that this treatment is helpful. In a Cochrane analysis, the conclusion was, based on current evidence, cranberry juice cannot currently be recommended for the prevention of UTIs.3 A natural product that appears to be more promising is the sugar D-mannose. Kranjčec et al. studied 308 women with acute UTI who had a history of recurrent UTI.4 All the women were treated for their symptomatic infection with ciprofloxacin (500 mg twice daily for 1 week). The women were allocated equally to three groups for 6 months: D-mannose 2 g daily, nitrofurantoin 50 mg daily, or no prophylaxis. About 60% of the women who received no prophylaxis had a UTI during the study period, compared with 20% in the nitrofurantoin group and 15% in the D-mannose group. The relative risk for D-mannose, compared with no prophylaxis, was 0.24 and for nitrofurantoin was 0.34 (P less than .0001), compared with no prophylaxis.

 

 

Made for migraines?

Migraine prophylaxis is challenging because all medications that are commonly used have side effects that often limit patient adherence. Tricyclic antidepressants (dry mouth, dizziness and weight gain), beta-blockers (fatigue, decreased exercise tolerance), valproate (weight gain and fatigue), and topiramate (parasthesias and mental slowing) all have troubling side effects. Riboflavin is a vitamin with evidence of effectiveness for migraine prophylaxis. It is extremely well tolerated. In a recent study in children with migraines, Talebian et al. studied 90 children with migraines who were randomized to three groups (200 mg of riboflavin a day, 100 mg of riboflavin a day, or placebo) after observation during a 1-month baseline period.5 There was a significant reduction in migraine frequency and duration in patients receiving 200 mg of riboflavin daily, compared with placebo. Rahimdel et al. published an interesting study comparing high-dose riboflavin with valproate for migraine prophylaxis. A total of 90 patients were randomized to receive 400 mg of riboflavin or 500 mg of valproate over a 12-month study.6 Both treatments resulted in marked reduction in frequency, duration, and severity of migraines (not statistically significantly different from each other). The reduction in migraine frequency for the riboflavin group was from 9.2 headache days per month to 2.4. The American Academy of Neurology rates the level of evidence for riboflavin as B.

Pearl

Horse chestnut, D-mannose, and riboflavin are safe alternative therapies that can be helpful for several common problems we see frequently in primary care.

References

1. Diehm C et al. Lancet. 1996;347(8997):292-4.

2. Pittler MH, Ernst E. Cochrane Database Syst Rev. 2012 Nov 14;11:CD003230.

3. Jepson RG et al. Cochrane Database Syst Rev. 2012;10:CD001321.

4. Kranjčec B et al. World J Urol. 2014 Feb;32(1):79-84.

5. Talebian A et al. Electron Physician. Feb 25;10(2):6279-85.

6. Rahimdel A et al. Electron Physician. 2015 Oct 19;7(6):1344-8.

Dr. Paauw is professor of medicine in the division of general internal medicine at the University of Washington, Seattle, and he serves as third-year medical student clerkship director at the University of Washington. Contact Dr. Paauw at dpaauw@uw.edu.

I practice in Seattle, where many of my patients are interested in natural treatment options. I am always puzzled that so many people assume that natural treatment options would be safer than prescription medications. Tsunamis, wildfires, flooding, and earthquakes are all natural and are deadly. There are certainly many natural poisons. There are a number of natural treatments that are very helpful, and recommending them are an important part of my practice. I want to share a few with you.

A useful vein pursuit?

Dr. Douglas S. Paauw, University of Washington, Seattle
Dr. Douglas S. Paauw

Case: A 60-year-old woman presents to clinic with increasing pain in her left leg. She had a deep vein thrombosis in her left leg 2 years ago, which involved a large portion of her superficial femoral vein. She has noticed edema over the past few weeks, and pain has been more severe over the past 3 months. On exam, varicosities on left lower extremity with grade 2+ edema. Duplex of the lower extremity does not show deep vein thrombosis, but does show a great deal of venous valvular incompetence. She has tried compression stockings for the past 2 weeks. What is the best treatment option?

A) Turmeric

B) Amitriptyline

C) Vitamin B12

D) Horse chestnut

The treatment option with positive data for this problem is horse chestnut. What is horse chestnut? Horse chestnut is a kind of tree, and the seed extract contains aescin which is believed to be the active ingredient. Diehm et al. studied horse chestnut seed extract (HCSE), compared with compression stockings and placebo for edema from chronic venous insufficiency in a study of 240 patients.1 Lower-leg volume decreased by 43 mL with HCSE, 46 mL with compression stockings, and increased by 9 mL with placebo (P less than .005 for HCSE and P less than .002 for compression). In a Cochrane review, HCSE was considered efficacious and safe for the short-term therapy for chronic venous insufficiency.2 Studies have shown both an improvement in pain as well as swelling in patients with chronic venous insufficiency.

The question of UTIs

Probably the most popular natural treatment for prevention and treatment of urinary tract infections (UTIs) in women is cranberry juice (or cranberry extract). Unfortunately, there is little evidence that this treatment is helpful. In a Cochrane analysis, the conclusion was, based on current evidence, cranberry juice cannot currently be recommended for the prevention of UTIs.3 A natural product that appears to be more promising is the sugar D-mannose. Kranjčec et al. studied 308 women with acute UTI who had a history of recurrent UTI.4 All the women were treated for their symptomatic infection with ciprofloxacin (500 mg twice daily for 1 week). The women were allocated equally to three groups for 6 months: D-mannose 2 g daily, nitrofurantoin 50 mg daily, or no prophylaxis. About 60% of the women who received no prophylaxis had a UTI during the study period, compared with 20% in the nitrofurantoin group and 15% in the D-mannose group. The relative risk for D-mannose, compared with no prophylaxis, was 0.24 and for nitrofurantoin was 0.34 (P less than .0001), compared with no prophylaxis.

 

 

Made for migraines?

Migraine prophylaxis is challenging because all medications that are commonly used have side effects that often limit patient adherence. Tricyclic antidepressants (dry mouth, dizziness and weight gain), beta-blockers (fatigue, decreased exercise tolerance), valproate (weight gain and fatigue), and topiramate (parasthesias and mental slowing) all have troubling side effects. Riboflavin is a vitamin with evidence of effectiveness for migraine prophylaxis. It is extremely well tolerated. In a recent study in children with migraines, Talebian et al. studied 90 children with migraines who were randomized to three groups (200 mg of riboflavin a day, 100 mg of riboflavin a day, or placebo) after observation during a 1-month baseline period.5 There was a significant reduction in migraine frequency and duration in patients receiving 200 mg of riboflavin daily, compared with placebo. Rahimdel et al. published an interesting study comparing high-dose riboflavin with valproate for migraine prophylaxis. A total of 90 patients were randomized to receive 400 mg of riboflavin or 500 mg of valproate over a 12-month study.6 Both treatments resulted in marked reduction in frequency, duration, and severity of migraines (not statistically significantly different from each other). The reduction in migraine frequency for the riboflavin group was from 9.2 headache days per month to 2.4. The American Academy of Neurology rates the level of evidence for riboflavin as B.

Pearl

Horse chestnut, D-mannose, and riboflavin are safe alternative therapies that can be helpful for several common problems we see frequently in primary care.

References

1. Diehm C et al. Lancet. 1996;347(8997):292-4.

2. Pittler MH, Ernst E. Cochrane Database Syst Rev. 2012 Nov 14;11:CD003230.

3. Jepson RG et al. Cochrane Database Syst Rev. 2012;10:CD001321.

4. Kranjčec B et al. World J Urol. 2014 Feb;32(1):79-84.

5. Talebian A et al. Electron Physician. Feb 25;10(2):6279-85.

6. Rahimdel A et al. Electron Physician. 2015 Oct 19;7(6):1344-8.

Dr. Paauw is professor of medicine in the division of general internal medicine at the University of Washington, Seattle, and he serves as third-year medical student clerkship director at the University of Washington. Contact Dr. Paauw at dpaauw@uw.edu.

Publications
Publications
Topics
Article Type
Sections
Disallow All Ads
Content Gating
No Gating (article Unlocked/Free)
Alternative CME
Disqus Comments
Default
Gate On Date
Wed, 12/31/1969 - 19:00
Un-Gate On Date
Wed, 12/31/1969 - 19:00
Use ProPublica
Hide sidebar & use full width
render the right sidebar.

Is carpal tunnel syndrome the tip of the iceberg?

Article Type
Changed
Wed, 11/20/2019 - 08:54

 

A 69-year-old man presents with increasing dyspnea on exertion. He has had recent orthopnea and paroxysmal nocturnal dyspnoea. He has a history of well controlled hypertension and hyperlipidemia. He takes the following medications: felodipine and atorvastatin. On exam, his blood pressure is 110/60 mm Hg, and his pulse is 90 beats per minute.

Dr. Douglas S. Paauw, University of Washington, Seattle
Dr. Douglas S. Paauw

A cardiac examination found normal heart sounds with no murmurs.

A chest examination found dullness to percussion at both bases and rales.

A chest x-ray showed bilateral effusions and mild pulmonary edema.

The brain natriuretic peptide test found a level of 1,300 picograms/mL.

An ECG found increased ventricular wall thickness, an ejection fraction of 32%, and normal aortic and mitral valves.

What history would be the most helpful in making a diagnosis?
 

A. History of prostate cancer

B. History of carpal tunnel syndrome

C. History of playing professional football

D. History of hyperlipidemia

E. History of ulcerative colitis

The correct answer here would be B. history of carpal tunnel syndrome (CTS). This patient has clinical heart failure, without a history of clinical ischemic disease. The differential diagnosis for causes of heart failure is long, with the most common causes being chronic hypertension and ischemic heart disease. Other common causes include chronic untreated sleep apnea and valvular heart disease.

This patient really does not have clear reasons for having clinical heart failure. His cardiovascular risk factors have been well controlled, and no valvular disease was found on ECG.

Several recent reports have raised the importance of a history of CTS significantly increasing the likelihood of amyloidosis being the cause of underlying heart failure.

CTS is such a common clinical entity that it is easy to not appreciate its presence as a clue to possible amyloid cardiomyopathy. Fosbøl et al. reported that a diagnosis of CTS was associated with a higher incidence of heart failure (hazard ratio, 1.54; CI, 1.45-1.64).1 They found a highly increased risk of amyloid (HR, 12.2) in patients who had surgery for CTS.

Sperry et al. found that over 10% of patients who underwent carpal tunnel release stained for amyloid on biopsy specimens, and that concomitant cardiac evaluation identified patients with cardiac involvement.2

Pinney et al. found that 48% of patients with transthyretin amyloidosis had a history of CTS.3

In a retrospective study of patients with wild-type transthyretin amyloid (253), patients with hereditary transthyretin amyloid (136), and asymptomatic gene carriers (77), participants were screened for a history of spinal stenosis and CTS.4 Almost 60% of the patients with amyloid had a history of CTS, and 11% had a history of spinal stenosis. Patients with CTS and hereditary amyloid had thicker interventricular septums, higher left ventricular mass, and lower Karnovsky index than those without CTS.

The diagnosis of CTS, especially in those who need surgery for treatment or have bilateral disease, should make us consider the possibility of underlying amyloidosis.

Pearl: In patients who have heart failure and a history of CTS, amyloidosis should be considered as a cause.

Dr. Paauw is professor of medicine in the division of general internal medicine at the University of Washington, Seattle, and serves as third-year medical student clerkship director at that university. Contact Dr. Paauw at imnews@mdedge.com.

References

1. Fosbøl EL et al. J Am Coll Cardiol. 2019;74:15-23.

2. Sperry BW et al. J Am Coll Cardiol. 2018 Oct 23;72(17):2040-50.

3. Pinney JH et al. J Am Heart Assoc. 2013 Apr 22;2(2):e000098.

4. Aus dem Siepen F et al. Clin Res Cardiol. 2019 Apr 5. doi: 10.1007/s00392-019-01467-1.
 

Publications
Topics
Sections

 

A 69-year-old man presents with increasing dyspnea on exertion. He has had recent orthopnea and paroxysmal nocturnal dyspnoea. He has a history of well controlled hypertension and hyperlipidemia. He takes the following medications: felodipine and atorvastatin. On exam, his blood pressure is 110/60 mm Hg, and his pulse is 90 beats per minute.

Dr. Douglas S. Paauw, University of Washington, Seattle
Dr. Douglas S. Paauw

A cardiac examination found normal heart sounds with no murmurs.

A chest examination found dullness to percussion at both bases and rales.

A chest x-ray showed bilateral effusions and mild pulmonary edema.

The brain natriuretic peptide test found a level of 1,300 picograms/mL.

An ECG found increased ventricular wall thickness, an ejection fraction of 32%, and normal aortic and mitral valves.

What history would be the most helpful in making a diagnosis?
 

A. History of prostate cancer

B. History of carpal tunnel syndrome

C. History of playing professional football

D. History of hyperlipidemia

E. History of ulcerative colitis

The correct answer here would be B. history of carpal tunnel syndrome (CTS). This patient has clinical heart failure, without a history of clinical ischemic disease. The differential diagnosis for causes of heart failure is long, with the most common causes being chronic hypertension and ischemic heart disease. Other common causes include chronic untreated sleep apnea and valvular heart disease.

This patient really does not have clear reasons for having clinical heart failure. His cardiovascular risk factors have been well controlled, and no valvular disease was found on ECG.

Several recent reports have raised the importance of a history of CTS significantly increasing the likelihood of amyloidosis being the cause of underlying heart failure.

CTS is such a common clinical entity that it is easy to not appreciate its presence as a clue to possible amyloid cardiomyopathy. Fosbøl et al. reported that a diagnosis of CTS was associated with a higher incidence of heart failure (hazard ratio, 1.54; CI, 1.45-1.64).1 They found a highly increased risk of amyloid (HR, 12.2) in patients who had surgery for CTS.

Sperry et al. found that over 10% of patients who underwent carpal tunnel release stained for amyloid on biopsy specimens, and that concomitant cardiac evaluation identified patients with cardiac involvement.2

Pinney et al. found that 48% of patients with transthyretin amyloidosis had a history of CTS.3

In a retrospective study of patients with wild-type transthyretin amyloid (253), patients with hereditary transthyretin amyloid (136), and asymptomatic gene carriers (77), participants were screened for a history of spinal stenosis and CTS.4 Almost 60% of the patients with amyloid had a history of CTS, and 11% had a history of spinal stenosis. Patients with CTS and hereditary amyloid had thicker interventricular septums, higher left ventricular mass, and lower Karnovsky index than those without CTS.

The diagnosis of CTS, especially in those who need surgery for treatment or have bilateral disease, should make us consider the possibility of underlying amyloidosis.

Pearl: In patients who have heart failure and a history of CTS, amyloidosis should be considered as a cause.

Dr. Paauw is professor of medicine in the division of general internal medicine at the University of Washington, Seattle, and serves as third-year medical student clerkship director at that university. Contact Dr. Paauw at imnews@mdedge.com.

References

1. Fosbøl EL et al. J Am Coll Cardiol. 2019;74:15-23.

2. Sperry BW et al. J Am Coll Cardiol. 2018 Oct 23;72(17):2040-50.

3. Pinney JH et al. J Am Heart Assoc. 2013 Apr 22;2(2):e000098.

4. Aus dem Siepen F et al. Clin Res Cardiol. 2019 Apr 5. doi: 10.1007/s00392-019-01467-1.
 

 

A 69-year-old man presents with increasing dyspnea on exertion. He has had recent orthopnea and paroxysmal nocturnal dyspnoea. He has a history of well controlled hypertension and hyperlipidemia. He takes the following medications: felodipine and atorvastatin. On exam, his blood pressure is 110/60 mm Hg, and his pulse is 90 beats per minute.

Dr. Douglas S. Paauw, University of Washington, Seattle
Dr. Douglas S. Paauw

A cardiac examination found normal heart sounds with no murmurs.

A chest examination found dullness to percussion at both bases and rales.

A chest x-ray showed bilateral effusions and mild pulmonary edema.

The brain natriuretic peptide test found a level of 1,300 picograms/mL.

An ECG found increased ventricular wall thickness, an ejection fraction of 32%, and normal aortic and mitral valves.

What history would be the most helpful in making a diagnosis?
 

A. History of prostate cancer

B. History of carpal tunnel syndrome

C. History of playing professional football

D. History of hyperlipidemia

E. History of ulcerative colitis

The correct answer here would be B. history of carpal tunnel syndrome (CTS). This patient has clinical heart failure, without a history of clinical ischemic disease. The differential diagnosis for causes of heart failure is long, with the most common causes being chronic hypertension and ischemic heart disease. Other common causes include chronic untreated sleep apnea and valvular heart disease.

This patient really does not have clear reasons for having clinical heart failure. His cardiovascular risk factors have been well controlled, and no valvular disease was found on ECG.

Several recent reports have raised the importance of a history of CTS significantly increasing the likelihood of amyloidosis being the cause of underlying heart failure.

CTS is such a common clinical entity that it is easy to not appreciate its presence as a clue to possible amyloid cardiomyopathy. Fosbøl et al. reported that a diagnosis of CTS was associated with a higher incidence of heart failure (hazard ratio, 1.54; CI, 1.45-1.64).1 They found a highly increased risk of amyloid (HR, 12.2) in patients who had surgery for CTS.

Sperry et al. found that over 10% of patients who underwent carpal tunnel release stained for amyloid on biopsy specimens, and that concomitant cardiac evaluation identified patients with cardiac involvement.2

Pinney et al. found that 48% of patients with transthyretin amyloidosis had a history of CTS.3

In a retrospective study of patients with wild-type transthyretin amyloid (253), patients with hereditary transthyretin amyloid (136), and asymptomatic gene carriers (77), participants were screened for a history of spinal stenosis and CTS.4 Almost 60% of the patients with amyloid had a history of CTS, and 11% had a history of spinal stenosis. Patients with CTS and hereditary amyloid had thicker interventricular septums, higher left ventricular mass, and lower Karnovsky index than those without CTS.

The diagnosis of CTS, especially in those who need surgery for treatment or have bilateral disease, should make us consider the possibility of underlying amyloidosis.

Pearl: In patients who have heart failure and a history of CTS, amyloidosis should be considered as a cause.

Dr. Paauw is professor of medicine in the division of general internal medicine at the University of Washington, Seattle, and serves as third-year medical student clerkship director at that university. Contact Dr. Paauw at imnews@mdedge.com.

References

1. Fosbøl EL et al. J Am Coll Cardiol. 2019;74:15-23.

2. Sperry BW et al. J Am Coll Cardiol. 2018 Oct 23;72(17):2040-50.

3. Pinney JH et al. J Am Heart Assoc. 2013 Apr 22;2(2):e000098.

4. Aus dem Siepen F et al. Clin Res Cardiol. 2019 Apr 5. doi: 10.1007/s00392-019-01467-1.
 

Publications
Publications
Topics
Article Type
Sections
Disallow All Ads
Content Gating
No Gating (article Unlocked/Free)
Alternative CME
Disqus Comments
Default
Use ProPublica
Hide sidebar & use full width
render the right sidebar.

Does this patient have bacterial conjunctivitis?

Article Type
Changed
Fri, 09/20/2019 - 10:02

 

A 54-year-old pharmacist with a history of gout, hypertension, and conjunctivitis presents for evaluation of pink eye in the summer. The morning before coming into the office, he noticed that his right eye was red and inflamed. He self-treated with saline washes and eye drops, but upon awakening the next day, he found his right eye to be crusted shut with surrounding yellow discharge. He has not had any changes to his vision but endorses a somewhat uncomfortable, “gritty” sensation. He reports no recent cough, nasal congestion, or allergies, and he has not been around any sick contacts. His blood pressure is 102/58 mm Hg, pulse is 76 bpm, and body mass index is 27.3 kg/m2. His eye exam reveals unilateral conjunctival injections but no hyperemia of the conjunctiva adjacent to the cornea. Mucopurulent discharge was neither found on the undersurface of the eyelid nor emerging from the eye. Which of the following is the best treatment for this patient’s condition?

A) Erythromycin 5 mg/gram ophthalmic ointment.

B) Ofloxacin 0.3% ophthalmic drops.

C) Antihistamine drops.

D) Eye lubricant drops.

E) No treatment necessary.

This patient is an adult presenting with presumed conjunctivitis. Because he is presenting in the summer without observed purulent discharge, his condition is unlikely to be bacterial. This patient does not need treatment, although eye lubricant drops could reduce his discomfort.

Nearly 1% of primary care office visits1 and 300 million in annual costs2 are spent evaluating and treating “pink eye.” After ruling out serious eye disease, clinicians need to determine which cases of suspected conjunctivitis are most likely to be bacterial to allow for judicious use of antibiotic eye drops. This is an important undertaking as most patients assume that antibiotics are needed.

McKenzie Momany

How do we know which history and clinical exam findings to lean on when attempting to categorize conjunctivitis as bacterial or not? If a patient reports purulent discharge, doesn’t that mean it is bacterial? Surprisingly, a systematic review published in 2016 by Narayana and McGee found that a patient’s self-report of “purulent drainage” is diagnostically unhelpful, but if a clinician finds it on exam, the likelihood of a bacterial etiology increases.3

Narayana and McGee analyzed three studies that enrolled a total of 281 patients with presumed conjunctivitis who underwent bacterial cultures. They then determined which findings increased the probability of positive bacterial culture. From strongest to weakest, the best indicators of a bacterial cause were found to be: complete redness of the conjunctival membrane obscuring tarsal vessels (the vessels visible on the inside of everted upper or lower eyelids) (likelihood ratio, 4.6), observed purulent discharge (LR, 3.9), matting of both eyes in the morning (LR, 3.6), and presence during winter/spring months (LR, 1.9). On the other hand, failure to observe a red eye at 20 feet (LR, 0.2), absence of morning gluing of either eye (LR, 0.3), and presentation during summer months (LR, 0.4) all decreased the probability of a bacterial cause. This review and different study by Stenson et al. unfortunately have conflicting evidence regarding whether the following findings are diagnostically helpful: qualities of eye discomfort (such as burning or itching), preauricular adenopathy, conjunctival follicles, and conjunctival papillae.3,4 Rietveld and colleagues found that a history of conjunctivitis decreased the likelihood of bacterial conjunctivitis.5

Dr. Douglas S. Paauw, University of Washington, Seattle
Dr. Doug Paauw


Ultimately, if the former indicators are kept in mind, primary care clinicians should be able to decrease the prescribing of topical antimicrobials to patients with non-bacterial conjunctivitis.

Pearl: The best indicators of a bacterial cause in patients with presumed conjunctivitis are complete redness of the conjunctival membrane obscuring tarsal vessels, observed purulent discharge, and matting of both eyes in the morning. Presentation during the summer months and having a history of conjunctivitis decreases the likelihood of bacterial conjunctivitis.

Ms. Momany is a fourth-year medical student at University of Washington, Seattle. Dr. Paauw is professor of medicine in the division of general internal medicine at the University of Washington and serves as third-year medical student clerkship director at that university. Contact Dr. Paauw at imnews@mdedge.com.

References

1. Azari AA and Barney NP. JAMA. 2013 Oct 23; 310(16):1721-9.

2. Smith AF and Waycaster C. BMC Ophthalmol. 2009 Nov 25. doi: 10.1186/1471-2415-9-13.

3) Narayana S and McGee S. Am J Med. 2015;128(11):1220-4.e1.

4) Stenson S et al. Arch Ophthalmol. 1982;100(8):1275-7.

5) Rietveld RP et al. BMJ. 2004 Jul 24;329(7459):206-10.

Publications
Topics
Sections

 

A 54-year-old pharmacist with a history of gout, hypertension, and conjunctivitis presents for evaluation of pink eye in the summer. The morning before coming into the office, he noticed that his right eye was red and inflamed. He self-treated with saline washes and eye drops, but upon awakening the next day, he found his right eye to be crusted shut with surrounding yellow discharge. He has not had any changes to his vision but endorses a somewhat uncomfortable, “gritty” sensation. He reports no recent cough, nasal congestion, or allergies, and he has not been around any sick contacts. His blood pressure is 102/58 mm Hg, pulse is 76 bpm, and body mass index is 27.3 kg/m2. His eye exam reveals unilateral conjunctival injections but no hyperemia of the conjunctiva adjacent to the cornea. Mucopurulent discharge was neither found on the undersurface of the eyelid nor emerging from the eye. Which of the following is the best treatment for this patient’s condition?

A) Erythromycin 5 mg/gram ophthalmic ointment.

B) Ofloxacin 0.3% ophthalmic drops.

C) Antihistamine drops.

D) Eye lubricant drops.

E) No treatment necessary.

This patient is an adult presenting with presumed conjunctivitis. Because he is presenting in the summer without observed purulent discharge, his condition is unlikely to be bacterial. This patient does not need treatment, although eye lubricant drops could reduce his discomfort.

Nearly 1% of primary care office visits1 and 300 million in annual costs2 are spent evaluating and treating “pink eye.” After ruling out serious eye disease, clinicians need to determine which cases of suspected conjunctivitis are most likely to be bacterial to allow for judicious use of antibiotic eye drops. This is an important undertaking as most patients assume that antibiotics are needed.

McKenzie Momany

How do we know which history and clinical exam findings to lean on when attempting to categorize conjunctivitis as bacterial or not? If a patient reports purulent discharge, doesn’t that mean it is bacterial? Surprisingly, a systematic review published in 2016 by Narayana and McGee found that a patient’s self-report of “purulent drainage” is diagnostically unhelpful, but if a clinician finds it on exam, the likelihood of a bacterial etiology increases.3

Narayana and McGee analyzed three studies that enrolled a total of 281 patients with presumed conjunctivitis who underwent bacterial cultures. They then determined which findings increased the probability of positive bacterial culture. From strongest to weakest, the best indicators of a bacterial cause were found to be: complete redness of the conjunctival membrane obscuring tarsal vessels (the vessels visible on the inside of everted upper or lower eyelids) (likelihood ratio, 4.6), observed purulent discharge (LR, 3.9), matting of both eyes in the morning (LR, 3.6), and presence during winter/spring months (LR, 1.9). On the other hand, failure to observe a red eye at 20 feet (LR, 0.2), absence of morning gluing of either eye (LR, 0.3), and presentation during summer months (LR, 0.4) all decreased the probability of a bacterial cause. This review and different study by Stenson et al. unfortunately have conflicting evidence regarding whether the following findings are diagnostically helpful: qualities of eye discomfort (such as burning or itching), preauricular adenopathy, conjunctival follicles, and conjunctival papillae.3,4 Rietveld and colleagues found that a history of conjunctivitis decreased the likelihood of bacterial conjunctivitis.5

Dr. Douglas S. Paauw, University of Washington, Seattle
Dr. Doug Paauw


Ultimately, if the former indicators are kept in mind, primary care clinicians should be able to decrease the prescribing of topical antimicrobials to patients with non-bacterial conjunctivitis.

Pearl: The best indicators of a bacterial cause in patients with presumed conjunctivitis are complete redness of the conjunctival membrane obscuring tarsal vessels, observed purulent discharge, and matting of both eyes in the morning. Presentation during the summer months and having a history of conjunctivitis decreases the likelihood of bacterial conjunctivitis.

Ms. Momany is a fourth-year medical student at University of Washington, Seattle. Dr. Paauw is professor of medicine in the division of general internal medicine at the University of Washington and serves as third-year medical student clerkship director at that university. Contact Dr. Paauw at imnews@mdedge.com.

References

1. Azari AA and Barney NP. JAMA. 2013 Oct 23; 310(16):1721-9.

2. Smith AF and Waycaster C. BMC Ophthalmol. 2009 Nov 25. doi: 10.1186/1471-2415-9-13.

3) Narayana S and McGee S. Am J Med. 2015;128(11):1220-4.e1.

4) Stenson S et al. Arch Ophthalmol. 1982;100(8):1275-7.

5) Rietveld RP et al. BMJ. 2004 Jul 24;329(7459):206-10.

 

A 54-year-old pharmacist with a history of gout, hypertension, and conjunctivitis presents for evaluation of pink eye in the summer. The morning before coming into the office, he noticed that his right eye was red and inflamed. He self-treated with saline washes and eye drops, but upon awakening the next day, he found his right eye to be crusted shut with surrounding yellow discharge. He has not had any changes to his vision but endorses a somewhat uncomfortable, “gritty” sensation. He reports no recent cough, nasal congestion, or allergies, and he has not been around any sick contacts. His blood pressure is 102/58 mm Hg, pulse is 76 bpm, and body mass index is 27.3 kg/m2. His eye exam reveals unilateral conjunctival injections but no hyperemia of the conjunctiva adjacent to the cornea. Mucopurulent discharge was neither found on the undersurface of the eyelid nor emerging from the eye. Which of the following is the best treatment for this patient’s condition?

A) Erythromycin 5 mg/gram ophthalmic ointment.

B) Ofloxacin 0.3% ophthalmic drops.

C) Antihistamine drops.

D) Eye lubricant drops.

E) No treatment necessary.

This patient is an adult presenting with presumed conjunctivitis. Because he is presenting in the summer without observed purulent discharge, his condition is unlikely to be bacterial. This patient does not need treatment, although eye lubricant drops could reduce his discomfort.

Nearly 1% of primary care office visits1 and 300 million in annual costs2 are spent evaluating and treating “pink eye.” After ruling out serious eye disease, clinicians need to determine which cases of suspected conjunctivitis are most likely to be bacterial to allow for judicious use of antibiotic eye drops. This is an important undertaking as most patients assume that antibiotics are needed.

McKenzie Momany

How do we know which history and clinical exam findings to lean on when attempting to categorize conjunctivitis as bacterial or not? If a patient reports purulent discharge, doesn’t that mean it is bacterial? Surprisingly, a systematic review published in 2016 by Narayana and McGee found that a patient’s self-report of “purulent drainage” is diagnostically unhelpful, but if a clinician finds it on exam, the likelihood of a bacterial etiology increases.3

Narayana and McGee analyzed three studies that enrolled a total of 281 patients with presumed conjunctivitis who underwent bacterial cultures. They then determined which findings increased the probability of positive bacterial culture. From strongest to weakest, the best indicators of a bacterial cause were found to be: complete redness of the conjunctival membrane obscuring tarsal vessels (the vessels visible on the inside of everted upper or lower eyelids) (likelihood ratio, 4.6), observed purulent discharge (LR, 3.9), matting of both eyes in the morning (LR, 3.6), and presence during winter/spring months (LR, 1.9). On the other hand, failure to observe a red eye at 20 feet (LR, 0.2), absence of morning gluing of either eye (LR, 0.3), and presentation during summer months (LR, 0.4) all decreased the probability of a bacterial cause. This review and different study by Stenson et al. unfortunately have conflicting evidence regarding whether the following findings are diagnostically helpful: qualities of eye discomfort (such as burning or itching), preauricular adenopathy, conjunctival follicles, and conjunctival papillae.3,4 Rietveld and colleagues found that a history of conjunctivitis decreased the likelihood of bacterial conjunctivitis.5

Dr. Douglas S. Paauw, University of Washington, Seattle
Dr. Doug Paauw


Ultimately, if the former indicators are kept in mind, primary care clinicians should be able to decrease the prescribing of topical antimicrobials to patients with non-bacterial conjunctivitis.

Pearl: The best indicators of a bacterial cause in patients with presumed conjunctivitis are complete redness of the conjunctival membrane obscuring tarsal vessels, observed purulent discharge, and matting of both eyes in the morning. Presentation during the summer months and having a history of conjunctivitis decreases the likelihood of bacterial conjunctivitis.

Ms. Momany is a fourth-year medical student at University of Washington, Seattle. Dr. Paauw is professor of medicine in the division of general internal medicine at the University of Washington and serves as third-year medical student clerkship director at that university. Contact Dr. Paauw at imnews@mdedge.com.

References

1. Azari AA and Barney NP. JAMA. 2013 Oct 23; 310(16):1721-9.

2. Smith AF and Waycaster C. BMC Ophthalmol. 2009 Nov 25. doi: 10.1186/1471-2415-9-13.

3) Narayana S and McGee S. Am J Med. 2015;128(11):1220-4.e1.

4) Stenson S et al. Arch Ophthalmol. 1982;100(8):1275-7.

5) Rietveld RP et al. BMJ. 2004 Jul 24;329(7459):206-10.

Publications
Publications
Topics
Article Type
Sections
Disallow All Ads
Content Gating
No Gating (article Unlocked/Free)
Alternative CME
Disqus Comments
Default
Use ProPublica
Hide sidebar & use full width
render the right sidebar.

How thin should we go?

Article Type
Changed
Mon, 08/26/2019 - 14:46

 

An 88-year-old man with hypertension, chronic obstructive pulmonary disease, and atrial fibrillation presents with severe cerebral palsy and is diagnosed with a non–ST-elevation MI. He is found to have 90% left anterior descending artery occlusion and receives a drug-eluting stent. His current medications include warfarin, tiotropium, amlodipine, aspirin, and lisinopril. What anticoagulant therapy should he receive?

Dr. Douglas S. Paauw, University of Washington, Seattle
Dr. Douglas S. Paauw

A) Clopidogrel, warfarin, and aspirin

B) Clopidogrel and aspirin

C) Clopidogrel and warfarin

D) Warfarin

E) Warfarin and aspirin

This issue comes up frequently with our patients with atrial fibrillation who are on anticoagulation, then have a coronary event and have a stent placed. What is the best approach to anticoagulation? I think for this patient adding clopidogrel, continuing warfarin, and stopping aspirin would be the best of the options presented.

Elderly patients have a higher risk of bleeding. They also have a greater chance of accumulating cardiovascular disease (atrial fibrillation, cardiac allograft vasculopathy, and valvular disease) that requires anticoagulation. Dewilde et al. studied the difference in bleeding risk in patients who were on oral anticoagulants who then underwent a percutaneous coronary intervention.1 Patients were assigned clopidogrel alone or clopidogrel plus aspirin in addition to their oral anticoagulant (warfarin). There was a significant increase in all-cause mortality in the patients who received clopidogrel plus aspirin (P = .027), and no significant difference in cardiac mortality between the two groups. There was a much higher risk of bleeding (44.4%) in the patients receiving triple therapy, compared with the double-therapy group (19.4%; P less than .0001).

In a large meta-analysis of over 7,000 patients by D’Ascenzo et al., there was no difference in thrombotic risk between double and triple therapy, and lower bleeding risk in patients who received double therapy.2

In a recently published article, Lopes et al. looked at the benefits and risks of antithrombotic therapy after acute coronary syndrome or percutaneous coronary intervention in patients with atrial fibrillation.3 The study included 4,614 patients, all of whom received a P2Y12 inhibitor. In addition, they received either apixaban or warfarin, and either aspirin or placebo. The patients who received apixaban had a lower risk of bleeding than those receiving warfarin (P less than .001), and those receiving aspirin had a higher risk than those receiving placebo (hazard ratio, 1.89; P less than .001). Patients using the combination of apixaban plus placebo had the lowest event rate per 100 years (16.8), followed by warfarin plus placebo (26.7), then apixaban plus aspirin (33.6), with warfarin plus aspirin having the highest event rate (49.1). The conclusion for the study was that regimens with apixaban without aspirin had less bleeding and hospitalizations without increased ischemic events, compared with regimens of warfarin with or without aspirin.

I think it is best to avoid aspirin in patients who are anticoagulated with warfarin, and likely this extends to Xa inhibitors as well.

Pearl: Avoid using triple anticoagulant therapy by eliminating aspirin.

Dr. Paauw is professor of medicine in the division of general internal medicine at the University of Washington, Seattle, and serves as third-year medical student clerkship director at the University of Washington. Contact Dr. Paauw at imnews@mdedge.com.

References

1. Dewilde WJ et al. Use of clopidogrel with or without aspirin in patients taking oral anticoagulant therapy and undergoing percutaneous coronary intervention: An open-label, randomised, controlled trial. Lancet. 2013 Mar 30;381(9872):1107-15.

2. D’Ascenzo F et al. Meta-analysis of randomized controlled trials and adjusted observational results of use of clopidogrel, aspirin, and oral anticoagulants in patients undergoing percutaneous coronary intervention. Am J Cardiol. 2015 May 1;115(9):1185-93.

3. Lopes RD et al. Antithrombotic therapy after acute coronary syndrome or PCI in atrial fibrillation. N Engl J Med. 2019 Apr 18;380(16):1509-24.

Publications
Topics
Sections

 

An 88-year-old man with hypertension, chronic obstructive pulmonary disease, and atrial fibrillation presents with severe cerebral palsy and is diagnosed with a non–ST-elevation MI. He is found to have 90% left anterior descending artery occlusion and receives a drug-eluting stent. His current medications include warfarin, tiotropium, amlodipine, aspirin, and lisinopril. What anticoagulant therapy should he receive?

Dr. Douglas S. Paauw, University of Washington, Seattle
Dr. Douglas S. Paauw

A) Clopidogrel, warfarin, and aspirin

B) Clopidogrel and aspirin

C) Clopidogrel and warfarin

D) Warfarin

E) Warfarin and aspirin

This issue comes up frequently with our patients with atrial fibrillation who are on anticoagulation, then have a coronary event and have a stent placed. What is the best approach to anticoagulation? I think for this patient adding clopidogrel, continuing warfarin, and stopping aspirin would be the best of the options presented.

Elderly patients have a higher risk of bleeding. They also have a greater chance of accumulating cardiovascular disease (atrial fibrillation, cardiac allograft vasculopathy, and valvular disease) that requires anticoagulation. Dewilde et al. studied the difference in bleeding risk in patients who were on oral anticoagulants who then underwent a percutaneous coronary intervention.1 Patients were assigned clopidogrel alone or clopidogrel plus aspirin in addition to their oral anticoagulant (warfarin). There was a significant increase in all-cause mortality in the patients who received clopidogrel plus aspirin (P = .027), and no significant difference in cardiac mortality between the two groups. There was a much higher risk of bleeding (44.4%) in the patients receiving triple therapy, compared with the double-therapy group (19.4%; P less than .0001).

In a large meta-analysis of over 7,000 patients by D’Ascenzo et al., there was no difference in thrombotic risk between double and triple therapy, and lower bleeding risk in patients who received double therapy.2

In a recently published article, Lopes et al. looked at the benefits and risks of antithrombotic therapy after acute coronary syndrome or percutaneous coronary intervention in patients with atrial fibrillation.3 The study included 4,614 patients, all of whom received a P2Y12 inhibitor. In addition, they received either apixaban or warfarin, and either aspirin or placebo. The patients who received apixaban had a lower risk of bleeding than those receiving warfarin (P less than .001), and those receiving aspirin had a higher risk than those receiving placebo (hazard ratio, 1.89; P less than .001). Patients using the combination of apixaban plus placebo had the lowest event rate per 100 years (16.8), followed by warfarin plus placebo (26.7), then apixaban plus aspirin (33.6), with warfarin plus aspirin having the highest event rate (49.1). The conclusion for the study was that regimens with apixaban without aspirin had less bleeding and hospitalizations without increased ischemic events, compared with regimens of warfarin with or without aspirin.

I think it is best to avoid aspirin in patients who are anticoagulated with warfarin, and likely this extends to Xa inhibitors as well.

Pearl: Avoid using triple anticoagulant therapy by eliminating aspirin.

Dr. Paauw is professor of medicine in the division of general internal medicine at the University of Washington, Seattle, and serves as third-year medical student clerkship director at the University of Washington. Contact Dr. Paauw at imnews@mdedge.com.

References

1. Dewilde WJ et al. Use of clopidogrel with or without aspirin in patients taking oral anticoagulant therapy and undergoing percutaneous coronary intervention: An open-label, randomised, controlled trial. Lancet. 2013 Mar 30;381(9872):1107-15.

2. D’Ascenzo F et al. Meta-analysis of randomized controlled trials and adjusted observational results of use of clopidogrel, aspirin, and oral anticoagulants in patients undergoing percutaneous coronary intervention. Am J Cardiol. 2015 May 1;115(9):1185-93.

3. Lopes RD et al. Antithrombotic therapy after acute coronary syndrome or PCI in atrial fibrillation. N Engl J Med. 2019 Apr 18;380(16):1509-24.

 

An 88-year-old man with hypertension, chronic obstructive pulmonary disease, and atrial fibrillation presents with severe cerebral palsy and is diagnosed with a non–ST-elevation MI. He is found to have 90% left anterior descending artery occlusion and receives a drug-eluting stent. His current medications include warfarin, tiotropium, amlodipine, aspirin, and lisinopril. What anticoagulant therapy should he receive?

Dr. Douglas S. Paauw, University of Washington, Seattle
Dr. Douglas S. Paauw

A) Clopidogrel, warfarin, and aspirin

B) Clopidogrel and aspirin

C) Clopidogrel and warfarin

D) Warfarin

E) Warfarin and aspirin

This issue comes up frequently with our patients with atrial fibrillation who are on anticoagulation, then have a coronary event and have a stent placed. What is the best approach to anticoagulation? I think for this patient adding clopidogrel, continuing warfarin, and stopping aspirin would be the best of the options presented.

Elderly patients have a higher risk of bleeding. They also have a greater chance of accumulating cardiovascular disease (atrial fibrillation, cardiac allograft vasculopathy, and valvular disease) that requires anticoagulation. Dewilde et al. studied the difference in bleeding risk in patients who were on oral anticoagulants who then underwent a percutaneous coronary intervention.1 Patients were assigned clopidogrel alone or clopidogrel plus aspirin in addition to their oral anticoagulant (warfarin). There was a significant increase in all-cause mortality in the patients who received clopidogrel plus aspirin (P = .027), and no significant difference in cardiac mortality between the two groups. There was a much higher risk of bleeding (44.4%) in the patients receiving triple therapy, compared with the double-therapy group (19.4%; P less than .0001).

In a large meta-analysis of over 7,000 patients by D’Ascenzo et al., there was no difference in thrombotic risk between double and triple therapy, and lower bleeding risk in patients who received double therapy.2

In a recently published article, Lopes et al. looked at the benefits and risks of antithrombotic therapy after acute coronary syndrome or percutaneous coronary intervention in patients with atrial fibrillation.3 The study included 4,614 patients, all of whom received a P2Y12 inhibitor. In addition, they received either apixaban or warfarin, and either aspirin or placebo. The patients who received apixaban had a lower risk of bleeding than those receiving warfarin (P less than .001), and those receiving aspirin had a higher risk than those receiving placebo (hazard ratio, 1.89; P less than .001). Patients using the combination of apixaban plus placebo had the lowest event rate per 100 years (16.8), followed by warfarin plus placebo (26.7), then apixaban plus aspirin (33.6), with warfarin plus aspirin having the highest event rate (49.1). The conclusion for the study was that regimens with apixaban without aspirin had less bleeding and hospitalizations without increased ischemic events, compared with regimens of warfarin with or without aspirin.

I think it is best to avoid aspirin in patients who are anticoagulated with warfarin, and likely this extends to Xa inhibitors as well.

Pearl: Avoid using triple anticoagulant therapy by eliminating aspirin.

Dr. Paauw is professor of medicine in the division of general internal medicine at the University of Washington, Seattle, and serves as third-year medical student clerkship director at the University of Washington. Contact Dr. Paauw at imnews@mdedge.com.

References

1. Dewilde WJ et al. Use of clopidogrel with or without aspirin in patients taking oral anticoagulant therapy and undergoing percutaneous coronary intervention: An open-label, randomised, controlled trial. Lancet. 2013 Mar 30;381(9872):1107-15.

2. D’Ascenzo F et al. Meta-analysis of randomized controlled trials and adjusted observational results of use of clopidogrel, aspirin, and oral anticoagulants in patients undergoing percutaneous coronary intervention. Am J Cardiol. 2015 May 1;115(9):1185-93.

3. Lopes RD et al. Antithrombotic therapy after acute coronary syndrome or PCI in atrial fibrillation. N Engl J Med. 2019 Apr 18;380(16):1509-24.

Publications
Publications
Topics
Article Type
Sections
Disallow All Ads
Content Gating
No Gating (article Unlocked/Free)
Alternative CME
Disqus Comments
Default
Use ProPublica
Hide sidebar & use full width
render the right sidebar.

Hyperthyroid? She sure doesn’t look like it

Article Type
Changed
Thu, 07/11/2019 - 22:11

A 50-year-old woman returns for routine follow-up. She has a 15-year history of hypothyroidism, pernicious anemia, and celiac disease. She has had some recent abdominal pain, but no changes in her bowel patterns, and she has not experienced any problems with chest pain, palpitations, or weakness recently. The only medication she is taking is levothyroxine 125 mcg. She has reported no recent weight loss, her blood pressure is 100/60 mm Hg, pulse is 66 beats per minute, temperature is 36.8 degrees Celsius, body mass index is 20, and she does not have a neck goiter. Her cardiac exam was normal and her neurological exam revealed no tremor. Her lab for thyroid-stimulating hormone (TSH) was less than 0.03, and her lab for free thyroxine (FT4) was 2.2, while her TSH level had been 1.4 a year ago. Her levothyroxine dose was decreased to 100 mcg/day, and her repeat lab for TSH, which occurred 12 weeks later, was still less than 0.03. What is the best explanation for why this patient’s labs look like hyperthyroidism, but this patient clinically does not appear to have hyperthyroidism?

A) She was initially given too much levothyroxine; her TSH response is lagging to dose reduction.

B) She has Graves’ disease.

C) She has acute thyroiditis.

D) She is taking extra thyroid hormone.

E) She is taking biotin.


Dr. Douglas S. Paauw, University of Washington, Seattle
Dr. Douglas S. Paauw

This patient has a history that includes multiple autoimmune diseases including hypothyroidism. It would be extremely unlikely that she would develop Graves' disease or develop acute thyroiditis in the setting of a gland that has been underfunctioning for years. She has no symptoms suggesting that she has hyperthyroidism, which makes taking more thyroid hormone than she is reporting less likely, although this could be possible. The TSH response can lag after dose adjustments of thyroid, but usually a 6-week interval is adequate. This patient’s testing was done 12 weeks after dose reduction making this very unlikely.

The cause for the labs that look like hyperthyroidism in this patient who appears clinically euthyroid is that she is taking biotin. Biotin (vitamin B7) has become a very popular supplement in the past few years for thin hair, brittle nails, and fatigue. The RDA for biotin is 30 mcg. It is widely available in high doses – 5,000-10,000 mcg – which are common doses for supplements.

Biotin has been used extensively as a key component of immunoassays. Streptavidin, a protein produced by the bacteria Streptomyces avidinii, binds biotin with an extremely high affinity, and this binding is utilized in a number of immunoassays, including the assays for thyroid hormone and TSH.1

 

 

High serum levels of biotin can make the assays inaccurate, with lower-than-actual TSH and higher-than-actual thyroid hormone levels. Multiple case reports have documented this happening clinically.1-3 I personally saw a case of this recently in my practice. Katzman and colleagues looked at the prevalence of biotin use in outpatients.4 They found that 7.7% were taking supplemental biotin, while 7.4% had levels of biotin in serum samples that were at a level that could interfere with biotin-based serum assays.

Theoretically, biotin can affect multiple other assays that use the streptavidin-biotin assay. The most concerning of these potential problems is with troponin assays. Biotin can falsely lower troponin assays and this can lead to missing the diagnosis of cardiac injury. The Food and Drug Administration released a warning about this and other biotin lab interactions in November 2017.5 Several studies have demonstrated that this effect can occur at serum levels achievable with available over-the-counter doses of biotin.6,7

Not all troponin assays are affected by high serum levels of biotin: The Gen 5 cTnT assay is the only troponin assay affected.7 I could not find any case reports that have been published where biotin had caused a clinical missed diagnosis with troponins.

Pearls

Consider biotin supplement use when you have patients whose labs look like hyperthyroidism, but clinically do not appear to be hyperthyroid.

Dr. Paauw is professor of medicine in the division of general internal medicine at the University of Washington, Seattle, and serves as third-year medical student clerkship director at the University of Washington. Contact Dr. Paauw at imnews@mdedge.com.

References

1. Charles S, Agrawal N, and Blum M. Erroneous thyroid diagnosis due to over-the-counter biotin. Nutrition 2019;57:257-8.

2. Elston MS et al. Factitious Graves’ disease due to a biotin immunoassay interference – a case and review of the literature. J Clin Endocrinol Metab 2016;101:3251-5.

3. Barbesino G. Misdiagnosis of Graves ’disease with apparent severe hyperthyroidism in a patient taking biotin megadoses. Thyroid 2016;26(6):860-3.

4. Katzman et al. Prevalence of biotin supplement usage in outpatients and plasma biotin concentrations in patients presenting to the emergency department. Clin Biochem. 2018 Sep;60:11-16.

5. “The FDA Warns that Biotin May Interfere with Lab Tests: FDA Safety Communication,” Nov. 28, 2017.

6. Trambas et al. Characterization of the scope and magnitude of biotin interference in susceptible Roche Elecsys competitive and sandwich immunoassays. Ann Clin Biochem. 2018 Mar;55(2):205-15.

7. Frame IJ et al. Susceptibility of cardiac troponin assays to biotin interference. Am J Clin Pathol. 2019 Apr 2;151(5):486-93.

Publications
Topics
Sections

A 50-year-old woman returns for routine follow-up. She has a 15-year history of hypothyroidism, pernicious anemia, and celiac disease. She has had some recent abdominal pain, but no changes in her bowel patterns, and she has not experienced any problems with chest pain, palpitations, or weakness recently. The only medication she is taking is levothyroxine 125 mcg. She has reported no recent weight loss, her blood pressure is 100/60 mm Hg, pulse is 66 beats per minute, temperature is 36.8 degrees Celsius, body mass index is 20, and she does not have a neck goiter. Her cardiac exam was normal and her neurological exam revealed no tremor. Her lab for thyroid-stimulating hormone (TSH) was less than 0.03, and her lab for free thyroxine (FT4) was 2.2, while her TSH level had been 1.4 a year ago. Her levothyroxine dose was decreased to 100 mcg/day, and her repeat lab for TSH, which occurred 12 weeks later, was still less than 0.03. What is the best explanation for why this patient’s labs look like hyperthyroidism, but this patient clinically does not appear to have hyperthyroidism?

A) She was initially given too much levothyroxine; her TSH response is lagging to dose reduction.

B) She has Graves’ disease.

C) She has acute thyroiditis.

D) She is taking extra thyroid hormone.

E) She is taking biotin.


Dr. Douglas S. Paauw, University of Washington, Seattle
Dr. Douglas S. Paauw

This patient has a history that includes multiple autoimmune diseases including hypothyroidism. It would be extremely unlikely that she would develop Graves' disease or develop acute thyroiditis in the setting of a gland that has been underfunctioning for years. She has no symptoms suggesting that she has hyperthyroidism, which makes taking more thyroid hormone than she is reporting less likely, although this could be possible. The TSH response can lag after dose adjustments of thyroid, but usually a 6-week interval is adequate. This patient’s testing was done 12 weeks after dose reduction making this very unlikely.

The cause for the labs that look like hyperthyroidism in this patient who appears clinically euthyroid is that she is taking biotin. Biotin (vitamin B7) has become a very popular supplement in the past few years for thin hair, brittle nails, and fatigue. The RDA for biotin is 30 mcg. It is widely available in high doses – 5,000-10,000 mcg – which are common doses for supplements.

Biotin has been used extensively as a key component of immunoassays. Streptavidin, a protein produced by the bacteria Streptomyces avidinii, binds biotin with an extremely high affinity, and this binding is utilized in a number of immunoassays, including the assays for thyroid hormone and TSH.1

 

 

High serum levels of biotin can make the assays inaccurate, with lower-than-actual TSH and higher-than-actual thyroid hormone levels. Multiple case reports have documented this happening clinically.1-3 I personally saw a case of this recently in my practice. Katzman and colleagues looked at the prevalence of biotin use in outpatients.4 They found that 7.7% were taking supplemental biotin, while 7.4% had levels of biotin in serum samples that were at a level that could interfere with biotin-based serum assays.

Theoretically, biotin can affect multiple other assays that use the streptavidin-biotin assay. The most concerning of these potential problems is with troponin assays. Biotin can falsely lower troponin assays and this can lead to missing the diagnosis of cardiac injury. The Food and Drug Administration released a warning about this and other biotin lab interactions in November 2017.5 Several studies have demonstrated that this effect can occur at serum levels achievable with available over-the-counter doses of biotin.6,7

Not all troponin assays are affected by high serum levels of biotin: The Gen 5 cTnT assay is the only troponin assay affected.7 I could not find any case reports that have been published where biotin had caused a clinical missed diagnosis with troponins.

Pearls

Consider biotin supplement use when you have patients whose labs look like hyperthyroidism, but clinically do not appear to be hyperthyroid.

Dr. Paauw is professor of medicine in the division of general internal medicine at the University of Washington, Seattle, and serves as third-year medical student clerkship director at the University of Washington. Contact Dr. Paauw at imnews@mdedge.com.

References

1. Charles S, Agrawal N, and Blum M. Erroneous thyroid diagnosis due to over-the-counter biotin. Nutrition 2019;57:257-8.

2. Elston MS et al. Factitious Graves’ disease due to a biotin immunoassay interference – a case and review of the literature. J Clin Endocrinol Metab 2016;101:3251-5.

3. Barbesino G. Misdiagnosis of Graves ’disease with apparent severe hyperthyroidism in a patient taking biotin megadoses. Thyroid 2016;26(6):860-3.

4. Katzman et al. Prevalence of biotin supplement usage in outpatients and plasma biotin concentrations in patients presenting to the emergency department. Clin Biochem. 2018 Sep;60:11-16.

5. “The FDA Warns that Biotin May Interfere with Lab Tests: FDA Safety Communication,” Nov. 28, 2017.

6. Trambas et al. Characterization of the scope and magnitude of biotin interference in susceptible Roche Elecsys competitive and sandwich immunoassays. Ann Clin Biochem. 2018 Mar;55(2):205-15.

7. Frame IJ et al. Susceptibility of cardiac troponin assays to biotin interference. Am J Clin Pathol. 2019 Apr 2;151(5):486-93.

A 50-year-old woman returns for routine follow-up. She has a 15-year history of hypothyroidism, pernicious anemia, and celiac disease. She has had some recent abdominal pain, but no changes in her bowel patterns, and she has not experienced any problems with chest pain, palpitations, or weakness recently. The only medication she is taking is levothyroxine 125 mcg. She has reported no recent weight loss, her blood pressure is 100/60 mm Hg, pulse is 66 beats per minute, temperature is 36.8 degrees Celsius, body mass index is 20, and she does not have a neck goiter. Her cardiac exam was normal and her neurological exam revealed no tremor. Her lab for thyroid-stimulating hormone (TSH) was less than 0.03, and her lab for free thyroxine (FT4) was 2.2, while her TSH level had been 1.4 a year ago. Her levothyroxine dose was decreased to 100 mcg/day, and her repeat lab for TSH, which occurred 12 weeks later, was still less than 0.03. What is the best explanation for why this patient’s labs look like hyperthyroidism, but this patient clinically does not appear to have hyperthyroidism?

A) She was initially given too much levothyroxine; her TSH response is lagging to dose reduction.

B) She has Graves’ disease.

C) She has acute thyroiditis.

D) She is taking extra thyroid hormone.

E) She is taking biotin.


Dr. Douglas S. Paauw

This patient has a history that includes multiple autoimmune diseases including hypothyroidism. It would be extremely unlikely that she would develop Graves' disease or develop acute thyroiditis in the setting of a gland that has been underfunctioning for years. She has no symptoms suggesting that she has hyperthyroidism, which makes taking more thyroid hormone than she is reporting less likely, although this could be possible. The TSH response can lag after dose adjustments of thyroid, but usually a 6-week interval is adequate. This patient’s testing was done 12 weeks after dose reduction making this very unlikely.

The cause for the labs that look like hyperthyroidism in this patient who appears clinically euthyroid is that she is taking biotin. Biotin (vitamin B7) has become a very popular supplement in the past few years for thin hair, brittle nails, and fatigue. The RDA for biotin is 30 mcg. It is widely available in high doses – 5,000-10,000 mcg – which are common doses for supplements.

Biotin has been used extensively as a key component of immunoassays. Streptavidin, a protein produced by the bacteria Streptomyces avidinii, binds biotin with an extremely high affinity, and this binding is utilized in a number of immunoassays, including the assays for thyroid hormone and TSH.1

 

 

High serum levels of biotin can make the assays inaccurate, with lower-than-actual TSH and higher-than-actual thyroid hormone levels. Multiple case reports have documented this happening clinically.1-3 I personally saw a case of this recently in my practice. Katzman and colleagues looked at the prevalence of biotin use in outpatients.4 They found that 7.7% were taking supplemental biotin, while 7.4% had levels of biotin in serum samples that were at a level that could interfere with biotin-based serum assays.

Theoretically, biotin can affect multiple other assays that use the streptavidin-biotin assay. The most concerning of these potential problems is with troponin assays. Biotin can falsely lower troponin assays and this can lead to missing the diagnosis of cardiac injury. The Food and Drug Administration released a warning about this and other biotin lab interactions in November 2017.5 Several studies have demonstrated that this effect can occur at serum levels achievable with available over-the-counter doses of biotin.6,7

Not all troponin assays are affected by high serum levels of biotin: The Gen 5 cTnT assay is the only troponin assay affected.7 I could not find any case reports that have been published where biotin had caused a clinical missed diagnosis with troponins.

Pearls

Consider biotin supplement use when you have patients whose labs look like hyperthyroidism, but clinically do not appear to be hyperthyroid.

Dr. Paauw is professor of medicine in the division of general internal medicine at the University of Washington, Seattle, and serves as third-year medical student clerkship director at the University of Washington. Contact Dr. Paauw at imnews@mdedge.com.

References

1. Charles S, Agrawal N, and Blum M. Erroneous thyroid diagnosis due to over-the-counter biotin. Nutrition 2019;57:257-8.

2. Elston MS et al. Factitious Graves’ disease due to a biotin immunoassay interference – a case and review of the literature. J Clin Endocrinol Metab 2016;101:3251-5.

3. Barbesino G. Misdiagnosis of Graves ’disease with apparent severe hyperthyroidism in a patient taking biotin megadoses. Thyroid 2016;26(6):860-3.

4. Katzman et al. Prevalence of biotin supplement usage in outpatients and plasma biotin concentrations in patients presenting to the emergency department. Clin Biochem. 2018 Sep;60:11-16.

5. “The FDA Warns that Biotin May Interfere with Lab Tests: FDA Safety Communication,” Nov. 28, 2017.

6. Trambas et al. Characterization of the scope and magnitude of biotin interference in susceptible Roche Elecsys competitive and sandwich immunoassays. Ann Clin Biochem. 2018 Mar;55(2):205-15.

7. Frame IJ et al. Susceptibility of cardiac troponin assays to biotin interference. Am J Clin Pathol. 2019 Apr 2;151(5):486-93.

Publications
Publications
Topics
Article Type
Sections
Disallow All Ads
Content Gating
No Gating (article Unlocked/Free)
Alternative CME
Disqus Comments
Default
Use ProPublica
Hide sidebar & use full width
render the right sidebar.

Patient-centered care in clinic

Article Type
Changed
Wed, 05/06/2020 - 12:20

Dr. Douglas S. Paauw
Dr. Douglas Paauw

Almost 30 years ago a young woman made an appointment to see me. I had just started my internal medicine practice and almost all the patients who saw me were new to me. I assumed she was establishing care with me. Her first words to me were “Hello Dr. Paauw, I would like to interview you to see if you will be a good fit as my doctor.” We talked for the 40-minute appointment time. I asked her about her health, her life, and what she wanted out of both. We shared with each other that we both were parents of young children. When the appointment was over, she said she would really like for me to be her doctor. She told me that the main thing she appreciated about me was that I listened, and that her previous physician never sat down at her appointments and often had his hand on the door handle for much of the visit. Physicians and patients both agree that compassionate care is essential for good patient care, yet about half of patients and 60% of doctors believe it is lacking in our medical system.1

 

Dr. Douglas Paauw
Here are some suggestions on how to help provide patient-centered care.

Remember the golden first minutes

When you step in the room and greet the patient, make sure the first few minutes are about connecting with a new patient or reconnecting with a patient who has visited your office in the past. I often start by asking the patient to give me an update on how they are doing. This lets me know what is important to them. I do not touch the computer until after this initial check-in.

Use the computer as a bond to strengthen your patient relationship

Many studies have shown patients find the computer gets between the doctor and patient. It is especially problematic if it breaks eye contact with the patient. People are less likely to share scary, sensitive, or embarrassing information if someone is looking at a computer and typing. As you look up tests, radiology reports, or consultant notes, let the patient in on what you are doing. Explain why you are searching in the record, and if it helps make an important point, show your findings to the patient. Offer to print out results, so they have something to carry with them.

Explain what you are looking for and what you find on the physical exam

Being a patient is scary. We all want reassurance that our fears are not true. When you find normal findings on exam, share those with the patient. Hearing “your heart sounds good, your pulses are strong” really helps patients. Explaining what we are doing when we examine is also helpful. Explain why you are feeling for lymph nodes in the neck, why we percuss the abdomen. Patients are often fascinated by getting a window into how we are thinking. I usually have medical students with me, which offers another avenue to explaining the how and why behind the exam. In asking and explaining to students, the patient is also taught why we do what we do.

 

 

Make sure that we cover what they are afraid of, not just what their symptom is

Patients come in not just to get symptom relief but to rest their mind from their fears of what it could be. I find it helpful to ask the patient what they think is the cause of the problem, or if they are worried about any specific diagnosis. With certain symptoms this is particularly important (for example, headaches, fatigue, or abdominal pain).

None of these suggestions are easy to do in busy, time-pressured clinic visits. I have found though that when patients feel cared about, listened to and can have their fears addressed they value our advice more, and less time is needed to negotiate the plan, as it has been developed together.

Dr. Paauw is professor of medicine in the division of general internal medicine at the University of Washington, Seattle, and serves as third-year medical student clerkship director at the University of Washington. Contact Dr. Paauw at fpnews@mdedge.com.

Reference

Lown BA et al. Health Aff (Millwood). 2011 Sep;30(9):1772-8.

Publications
Topics
Sections

Dr. Douglas S. Paauw
Dr. Douglas Paauw

Almost 30 years ago a young woman made an appointment to see me. I had just started my internal medicine practice and almost all the patients who saw me were new to me. I assumed she was establishing care with me. Her first words to me were “Hello Dr. Paauw, I would like to interview you to see if you will be a good fit as my doctor.” We talked for the 40-minute appointment time. I asked her about her health, her life, and what she wanted out of both. We shared with each other that we both were parents of young children. When the appointment was over, she said she would really like for me to be her doctor. She told me that the main thing she appreciated about me was that I listened, and that her previous physician never sat down at her appointments and often had his hand on the door handle for much of the visit. Physicians and patients both agree that compassionate care is essential for good patient care, yet about half of patients and 60% of doctors believe it is lacking in our medical system.1

 

Dr. Douglas Paauw
Here are some suggestions on how to help provide patient-centered care.

Remember the golden first minutes

When you step in the room and greet the patient, make sure the first few minutes are about connecting with a new patient or reconnecting with a patient who has visited your office in the past. I often start by asking the patient to give me an update on how they are doing. This lets me know what is important to them. I do not touch the computer until after this initial check-in.

Use the computer as a bond to strengthen your patient relationship

Many studies have shown patients find the computer gets between the doctor and patient. It is especially problematic if it breaks eye contact with the patient. People are less likely to share scary, sensitive, or embarrassing information if someone is looking at a computer and typing. As you look up tests, radiology reports, or consultant notes, let the patient in on what you are doing. Explain why you are searching in the record, and if it helps make an important point, show your findings to the patient. Offer to print out results, so they have something to carry with them.

Explain what you are looking for and what you find on the physical exam

Being a patient is scary. We all want reassurance that our fears are not true. When you find normal findings on exam, share those with the patient. Hearing “your heart sounds good, your pulses are strong” really helps patients. Explaining what we are doing when we examine is also helpful. Explain why you are feeling for lymph nodes in the neck, why we percuss the abdomen. Patients are often fascinated by getting a window into how we are thinking. I usually have medical students with me, which offers another avenue to explaining the how and why behind the exam. In asking and explaining to students, the patient is also taught why we do what we do.

 

 

Make sure that we cover what they are afraid of, not just what their symptom is

Patients come in not just to get symptom relief but to rest their mind from their fears of what it could be. I find it helpful to ask the patient what they think is the cause of the problem, or if they are worried about any specific diagnosis. With certain symptoms this is particularly important (for example, headaches, fatigue, or abdominal pain).

None of these suggestions are easy to do in busy, time-pressured clinic visits. I have found though that when patients feel cared about, listened to and can have their fears addressed they value our advice more, and less time is needed to negotiate the plan, as it has been developed together.

Dr. Paauw is professor of medicine in the division of general internal medicine at the University of Washington, Seattle, and serves as third-year medical student clerkship director at the University of Washington. Contact Dr. Paauw at fpnews@mdedge.com.

Reference

Lown BA et al. Health Aff (Millwood). 2011 Sep;30(9):1772-8.

Dr. Douglas S. Paauw
Dr. Douglas Paauw

Almost 30 years ago a young woman made an appointment to see me. I had just started my internal medicine practice and almost all the patients who saw me were new to me. I assumed she was establishing care with me. Her first words to me were “Hello Dr. Paauw, I would like to interview you to see if you will be a good fit as my doctor.” We talked for the 40-minute appointment time. I asked her about her health, her life, and what she wanted out of both. We shared with each other that we both were parents of young children. When the appointment was over, she said she would really like for me to be her doctor. She told me that the main thing she appreciated about me was that I listened, and that her previous physician never sat down at her appointments and often had his hand on the door handle for much of the visit. Physicians and patients both agree that compassionate care is essential for good patient care, yet about half of patients and 60% of doctors believe it is lacking in our medical system.1

 

Dr. Douglas Paauw
Here are some suggestions on how to help provide patient-centered care.

Remember the golden first minutes

When you step in the room and greet the patient, make sure the first few minutes are about connecting with a new patient or reconnecting with a patient who has visited your office in the past. I often start by asking the patient to give me an update on how they are doing. This lets me know what is important to them. I do not touch the computer until after this initial check-in.

Use the computer as a bond to strengthen your patient relationship

Many studies have shown patients find the computer gets between the doctor and patient. It is especially problematic if it breaks eye contact with the patient. People are less likely to share scary, sensitive, or embarrassing information if someone is looking at a computer and typing. As you look up tests, radiology reports, or consultant notes, let the patient in on what you are doing. Explain why you are searching in the record, and if it helps make an important point, show your findings to the patient. Offer to print out results, so they have something to carry with them.

Explain what you are looking for and what you find on the physical exam

Being a patient is scary. We all want reassurance that our fears are not true. When you find normal findings on exam, share those with the patient. Hearing “your heart sounds good, your pulses are strong” really helps patients. Explaining what we are doing when we examine is also helpful. Explain why you are feeling for lymph nodes in the neck, why we percuss the abdomen. Patients are often fascinated by getting a window into how we are thinking. I usually have medical students with me, which offers another avenue to explaining the how and why behind the exam. In asking and explaining to students, the patient is also taught why we do what we do.

 

 

Make sure that we cover what they are afraid of, not just what their symptom is

Patients come in not just to get symptom relief but to rest their mind from their fears of what it could be. I find it helpful to ask the patient what they think is the cause of the problem, or if they are worried about any specific diagnosis. With certain symptoms this is particularly important (for example, headaches, fatigue, or abdominal pain).

None of these suggestions are easy to do in busy, time-pressured clinic visits. I have found though that when patients feel cared about, listened to and can have their fears addressed they value our advice more, and less time is needed to negotiate the plan, as it has been developed together.

Dr. Paauw is professor of medicine in the division of general internal medicine at the University of Washington, Seattle, and serves as third-year medical student clerkship director at the University of Washington. Contact Dr. Paauw at fpnews@mdedge.com.

Reference

Lown BA et al. Health Aff (Millwood). 2011 Sep;30(9):1772-8.

Publications
Publications
Topics
Article Type
Sections
Disallow All Ads
Content Gating
No Gating (article Unlocked/Free)
Alternative CME
Disqus Comments
Default
Use ProPublica
Hide sidebar & use full width
render the right sidebar.